Download as docx, pdf, or txt
Download as docx, pdf, or txt
You are on page 1of 135

..

SSIP JULY – AUGUST 2021


MATHEMATICS
PARTICIPANTS GUIDE

1
© COPYRIGHT

This work is protected by the Copyright Act 98 of 1978. No part of this work may be reproduced or
transmitted in any form or by any means, electronic or mechanical, including photocopying,
recording or by any information storage and retrieval system, without permission in writing from
Matthew Goniwe School of Leadership and Governance.

Whilst every effort has been made to ensure that the information published in this work is accurate,
Matthew Goniwe School of Leadership and Governance takes no responsibility for any loss or
damage suffered by any person as a result of the reliance upon the information contained therein.

2
TABLE OF CONTENTS
NO. PREAMBLE PAGE

I Foreword 4

II Purpose 5

III SSIP Aims/goals 4

IV SSIP Objectives 4

V Learning Assumed to be in place 4

VI Target Audience 5

VII Notional Hours 5

VIII Course Design and Assessment Strategy 5

IX Course outline/Map 5-7

X Table of Icons 7

XI Table of Acronyms and Abbreviations 8

XII Term ATP 9

XIII Course Timetable 10

MODULE 1: STATISTICS

Unit 1 Measure of central tendency and measure of dispersion 12 - 36

Unit 2 Grouped data 37 - 58

Unit 3 Univariate and Bivariate Data 59 - 74

EXAM TYPE QUESTIONS

MODULE 2 : PROBABILITY

Unit 1 Mutually , exclusive events, Complementary events 76 - 85

UNIT 2 Independent events, Tree diagram and contingence table 86 - 93

UNIT 3 Fundamental counting Principle 94 - 107

MODULE 3:FINANCIAL MATHEMATICS

Unit 1 Growth, Decay and Annuities 109 - 132

EXAM TYPE QUESTIONS 130 - 132

Summary 133

Conclusion 133

Reference 133

3
I. FOREWORD

The e-SSIP for teacher development is the initiative of the Gauteng Department of Education (GDE)
in collaboration with Matthew Goniwe School of Leadership and Governance (MGSLG). The project
involves training of teachers on weekends, running from Fridays to Sundays and was started in 2016
academic year.

II. PURPOSE

The purpose of this manual is to provide teachers with subject knowledge, content simplification,
teaching skills and tips and integration of technology (ICT skills) in the classroom.

III. SSIP AIMS/GOALS

The SSIP programme aims at professional development for Grade 10-12 teachers in the application
of effective teaching and reflective practice to improve learner performance on the identified Grade
12 examinable topics. The overall goal for SSIP is to provide teachers with professional expertise,
tools and skills to spot student learning difficulties and decide on the course of action.

SSIP came about as result of the diagnostic needs that are identified through the end of the year NSC
examination student learning data. In response to this design and development of teaching
resources are developed to train teachers on the learner needs.

The four interconnected outcomes that drive the professional development activities for SSIP are:

 Enhancing Teachers knowledge: deep understanding of subject matter knowledge and


students ideas on the content
 Enhancing quality teaching and assessment for learning: effective instructional approaches
that teachers may use to ensure improved understanding by most learners.
 Developing ICT integration skills :Use of ICT to improve teaching and learning
 Building professional learning communities: allow teachers to start collaborating and form
professional networks in non-formal settings in context of their schools

IV. SSIP OBJECTIVES

By the end of the workshop teachers should be able to:

 have mastered and understood all aspects related to Probability, Statistics and Financial
Mathematics within the FET Mathematics Curriculum

 utilise ICT integration and encourage interactive lessons in teaching and learning

V. LEARNING ASSUMED TO BE IN PLACE

Majority of the Mathematics educators identified, acquired qualifications in Mathematics, Sciences


and Commercial Subjects however most the teachers were CAPS oriented to teach the subject. It
was evident from the monitoring and diagnostic reports that those teachers lack pedagogy or
subject knowledge to teach the subject efficiently and effectively.

4
VI. TARGET AUDIENCE

Teachers were identified through the 2020 NSC results, diagnostics report, and needs analyses of the
teacher in the Integrated Quality Management System (IQMS).

VII. NOTIONAL HOURS:

The time allocation in Mathematics according to CAPS and NPPPPR is 4, 5 hours per week.

VIII. COURSE DESIGN AND ASSESSMENT STRATEGY

 The envisaged course focuses on three modules with at least two units each viz.

 Module 1: Statistics

 Module 2: Probability

 Module 3: Financial Mathematics

 Teachers will be subjected to the variety of content knowledge, formative activities to


consolidate the content learnt, Pre-test at the beginning of each workshop session to further
verify content gaps and post-test in the last session.

IX. COURSE OUTLINE/ MAP

Module 1: Statistics

Objectives/Outcomes Units

 Interpret univariate numerical data in order to Unit 1:


determine measures of central tendency (mean,
Measure of central tendency and measure of
median, mode in ungrouped and grouped data.
dispersion
 Five number summary
 Box and whisker diagrams Unit 2:

 Measures of dispersion (range, percentiles, and Grouped data


quartiles.

 Histograms and frequency polygons; Variance


Unit 3
and standard deviation of ungrouped data
Ogives( Cumulative frequency curves) Use statistical summaries , scatterplots ,

 Symmetric and skewed data. regression ( Least squares regression line) and

 Identification of outliers. correlation to analyse and make meaningful

5
 Use statistical summaries , scatterplots , comments on the context associated with
regression ( Least squares regression line) and given bivariate data, including interpolation ,
correlation to analyse and make meaningful extrapolation, and discussion on skewness
comments on the context associated with given
bivariate data, including interpolation ,
extrapolation and discussion on skewness
Module 2 : Probability

Objectives/Outcomes Units

 Compare the relative frequency of an Unit 1:


experimental the theoretical probability of Compare the relative frequency of an
the outcome. experimental the theoretical probability of
 Venn diagrams as an aid to solving the outcome.
probability. Venn diagrams as an aid to solving probability.
 Mutually exclusive events and Mutually exclusive events and complementary
complementary events. events.
 The identity of any two events The identity of any two events

P ( A∨B )=P ( A ) + P ( B )−P( A∧B) P ( A∨B )=P ( A ) + P ( B )−P( A∧B)


 Independent and dependent events. Unit 2

 Two-way contingency table. Independent and dependent events.


 The product rule for independent events. Two-way contingency table.
P ( A∧B )=P ( A )× P(B) The product rule for independent events.
 Venn diagram , tree diagram and other P ( A∧B )=P ( A )× P(B)
techniques to solve probability problems Venn diagram , tree diagram and other
where events are not necessarily techniques to solve probability problems
independent where events are not necessarily independent
 Probability problems using fundamental Unit 3
counting principle
Probability problems using fundamental
counting principle

Module 3 : Financial Mathematics

6
Objectives/Outcomes Unit 1

 Use simple and compound growth formula to Simple and compound interest
solve problems.
Inflation and Hire purchase
 Annual interest hire purchase and inflation.
 Different periods of compound growth and Effective and nominal Rate
decay.
Depreciation
 Effective and nominal interest rates.
 Solve problems using present and future value Annuities
annuities.
 Calculate the value of n in the formulae A=P¿
or A=P¿ .
 Critically analyse investments and loan options.

X. TABLE OF ICONS TO BE USED IN THIS MANUAL

Discussion

Group Activity

Individual Activity

Study Tips

Notes

Ice Breaker

Time

Tools

XI. TABLE OF ACRONYMS AND ABBREVIATIONS

7
Acronym Definition

ATP Annual Teaching Plan


CAPS Curriculum and Assessment Policy Statement

ICT Information and Communication Technology


LP Lesson Plan

FG Facilitator’s Guide
NPPPPR National Policy Pertaining to Programme and Promotion requirements

PG Participant’s Guide
FS Fact Sheet

PPT PowerPoint Presentation


TPACK Technological, Pedagogical, Content and Knowledge

TS Training Session

XII. TERM 3 ANNUAL TEACHING PLAN

8
GRADE 12 MATHEMATICS ATP 2021
Date %
DATE Completed
TOPIC CONTENT F ASSESSMENT Completed

TERM 3 2 TASKS TERM 3

 Make use of logarithms to calculate the value of n


Week 1 , the time period, in the equations
26/7 – 30/7 Financial  Critically analyse investment and loan options 81%
Mathematics and make informed decisions as to best option(s)
(including pyramid).
 Histograms
Week 2  Frequency polygons
02/8 – 06/8 Statistics  Ogives (cumulative frequency curves) 85%
 Variance and standard deviation of ungrouped
data
Week 3  Symmetric and skewed data
 Identification of outliers
09/8 –13/8 Statistics 88%
 Revise symmetric and skewed data.
(4 days)
 Use statistical summaries, scatterplots, TEST
regression (in particular the least squares
Week 4 regression line) and correlation to analyse and
16/8 – 20/8 make meaningful comments on the context SBA marks:15% 92%
Statistics
associated with given bivariate data, including
interpolation, extrapolation and discussions on
skewness.
 Revised the addition rule for mutually exclusive  
events: P ( A∨B )=PA+ P (B)
 The complementary rule:
P ( not A ) =1−P( A) and the identity
Week 5 P ( A∨B )=P ( A ) + P ( B )−P( A∧B)
23/8 - 27/8 Counting and  Identify dependents and independents events 96%
Probability and the product rule for independent events:
P ( A∧B )=P ( A ) × P( B)
 The use of Venn diagrams to solve probability
problems, deriving and applying formulae for any
three events A, B and C
in a sample space S.
 Use tree diagrams for the probability of
consecutive or simultaneous events which are
not necessarily independent.
 Probability problems using Venn diagrams, tree
Week 6 Counting and diagrams, two-way contingency tables
100%
30/8 – 03/9 Probability and other techniques to solve probability
problems (where events are not necessarily
independent).
 Apply the fundamental counting principle to solve
probability problems.
Week 7 PRELIMINARY
06/9 – 10/9 Prelim exam EXAMINATIONS
Week 8
13/9 – 17/9 Prelim exam SBA marks:25%
Week 9 %
20/9 – 24/9 Prelim exam
(4 days)
Week 10
Prelim exam
27/9 – 01/10
END OF TERM 3 SCHOOLS CLOSES ON THE 01/10/2021

9
XIII. COURSE TIMETABLE

DAY 1

TIME ACTIVITY

12:00 – 12:30 Plenary Session: Opening and Welcome, registration and Other Logistics

12:35 – 13:35 TS 1: Pre-Test

14:00 – 15:00 Introduction of topic 1

15:00 – 15 :30 Closure

DAY 2

TIME ACTIVITY

08:00 – 10:30 TS 2: Module 1 – Unit 1

10:30 – 11:00 Tea Break

11:00 – 13:00 TS 3: Module 1 – Unit 2

13:00 – 14:00 Lunch

14:00 – 15:30 TS 4: Module 1 – Unit 3

DAY 3

TIME ACTIVITY

08:00 – 09:00 TS 8: Module 2– Unit 1

09:00 – 10:30 TS 9: Module 2 – Unit 2

10:30 – 11:00 Tea Break

11:00 – 11:45 TS 10: Module 2 – Unit 3

11:45 – 12:45 TS 11: Post Test

12:45 – 13:00 Closing Session:


Closing Remarks

10
MODULE 1: STATISTICS

INTRODUCTION 
In this module participants will look at the concept of Statistics with specific focus on: 
 measure of central tendency  
 Measure of dispersion  
 Statistic Variables and organization of the data  
 Describing data by tables  
 Describing data by graphs 

OVERVIEW 
In this topic, participants will work with complex projects in familiar and unfamiliar contexts 
SPECIFIC OBJECTIVES  

When participants complete this module, they should be able to: 


 Interpret univariate numerical data to determine measures of central tendency  
(mean, median, mode in ungrouped and grouped data). 
 Determine the Five number summary 
 Box and whisker diagrams 
 Measures of dispersion (range, percentiles and quartiles). 
 Histograms and frequency polygons; Variance and standard deviation of
ungrouped data Ogives (Cumulative frequency curves) 
 Symmetric and skewed data. 
 Identification of outliers. 
 Use statistical summaries, scatter plots, regression (Least squares regression line)
and correlation to analyse and make meaningful comments on the context associated
with given bivariate data, including interpolation, extrapolation, and discussion on
skewness 
 
1.LIST OF CONCEPTS  
Acronym    Definition 
Data  :  The word data is the plural of the word datum which means “a piece of
information”.  So, data are pieces of information. 
Organising and :   Determining measures of CENTRAL TENDENCY (mean, median, mode)
summarising data  (PART 1); determining MEASURES OF DISPERSION (range,
extremes, outliers) (PART2), Standard deviation and regression 

Representing data  :  Histograms; Frequency Polygon, pie charts, Box and Whisker, Ogive,
Scatter plots. 
Interpreting data  :  Critically reading and interpreting two sets of data represented in a variety
of graphs. 
Analysing data  :  critically analysing data by answering questions related to data collection
methods, summaries of data, sources of error and bias in the data 
Reporting data  :  drawing conclusions about the data; making predictions based on the
data; making comparisons between two sets of data; identifying sources
of error and bias in the data; choosing  appropriate summary statistics
(mean, median, mode, range) for the data and discussing the role of
extremes and outliers in the data  
Ungrouped data    Date is put into classes or within a certain interval.  
Grouped data    It numeric or items without a range or interval. 

11
 
UNIT 1:  Measure of central tendency and measure of dispersion. Box and Whisker  

INTRODUCTION  
 
In this unit participants will look at the concept of measure of central tendency and measure of
dispersion in ungrouped data. 

LEARNING OUTCOME 

At the end of this Unit, participants should be able to: 


• Revise the language of data handling. 
• Determine measures of central tendency of lists of data.  
• Determine quartiles and the five-number-summary  
• Determine percentiles 
 • Determine measures of dispersion (range and inter-quartile range) 
 • Illustrate the five-number summary with a box and whisker diagram 

STEM AND LEAF 

To analyse data, sometimes we need to sort the data in ascending order, we use the stem and
leaf method. (In ungrouped data.)
 
A stem-and-leaf is a table used to display data. The 'stem' is on the left displays the first digit or
digits. The 'leaf' is on the right and displays the last digit. 
 
EXAMPLE 1 
Given the data  STE LEA              
15  27  8   17 13 22  M  F 
24 24 25 13 36 32  32 32 28 43 7 0  7  8            
1  3  3 5 7        

2  2    4 4  5 7  8  


   
3  2  2 2     6    

4  3           

12
Activity 1.  
The data in the table below represents the MATHEMATICS test marks of the learners out
of 50 marks. Complete the table below:  

Data 1 Ungrouped  ORDERED DATA  


26      8        17        16        28    Stem  Leaf 
32      25      14        45        41        
39      49     33         22        27  
   
22      6       22         36        46      
25      18     21         6          11       
   
   
   

MEASURES OF CENTRAL TENDENCY OF LISTS OF DATA. 

 
 
 An average or measure of central tendency is a single number which is used to represent a collection of
numerical data.  The commonly used averages are the mean, median and mode.  
 When we calculate the mean, median and mode we are finding the value of a typical item in a data set. 

 
 The symbol Σ (sigma) tells us to
add all the values in the data set. 
  The symbol n is the total number of
items in the data set 

UNGROUPED DATA

x=
∑ x = x 1 + x 2+ x 3 +… … …+ x n
n n
GROUPED DATA ESTIMATED MEAN

mi × f i ( m1 × f 1 ) + ( m2 × f 2 )+ ( m3 × f 3 ) + … … … . ( mn × f n )
x= =
n n

13
EXAMPLE 2: 
 8 participants in bike race had the following finishing times in minutes: 28,22,26,29,21,23,24,50. 

 a) Calculate the mean of the finishing time for the bikers. 
 b) What does the mean tell us about the finishing time?  
c) Use your scientific calculator to determine the mean finishing time. 
SOLUTIONS: 
 a)  
  ∑x  = 28   +   22   +   26    +   29    +   21   +   23   +   24   +   50 
              = 223 min 
There are 8 terms in the data set, so n = 8  
 Mean: 

x=
∑ x   
n
223
x=
8
x=27,88

 The mean time travelled by the 8 bickers is 27,88. 

b) The mean finishing time tells us that if all the finishing time are added together and shared out equally,
each biker would finish the race in 27,88 min.  

ONE of these bikers finished after 50 minutes.  This biker’s finishing time is an outlier affect the value of the
mean and make it larger than it should be if only the 7 bikers were to be considered. The mean would be ≈
24,71 less than 27,88  
c) The key sequences for the Casio fx-82ZA PLUS and the Sharp EL-W535SA are as follows:  
28   +   22   +   26    +   29    +   21   +   23   +   24   +   50 
 CASIO Calculator (fx-82ZA PLUS):   
Press Mode than STAT then 1-VAR 

Enter all data one at the time pressing = after each entry. 


Press AC button 
Press Shift then STAT then Var 
To calculate the mean press 2:  x then =  
 

14
SHARP Calculator (EL-W535SA) 
[Press Mode than 1-STAT then 0-SD 

Enter all data one at the time in the (x-column) pressing = after each entry. 
Press ON/C button 
Press Alpha then 8-STAT  
then 0-STATISTICS read off the value: x
 
 Both calculators give the value : x = 27,875... ≈ 27,88 minutes. 
EXAMPLE 3. 
Lebo recorded the amount of data (in MB) that she had used on each of the first 15 days in
April. The information is shown below. 

26 13 3 18 12 34 24 58 16
10 15 69 20 17 40

1.Calculate the: 

a. Mean for the data set 

b. Calculate the maximum total amount of data that Lebo must use for

remainder of the month if she wishes for the overall mean of April to be 80% of the mean for the first 15
days. 

Solution:  

a) Using a calculator: value x=25

b) let the remaining days be :y:  ∑xi=375 for the first 15 days of April month. 

375+ ∑ y i
 25 ×80 %=
30

15
∑ yi =225
The total is 225 MB for the remaining days. 

EXAMPLE 4

A representative sample of Secondary Schools in South Africa took part in the 2009 Census@School.
The mean number of schools per province for 8 of the 9 provinces (Mpumalanga is excluded) was
66.

a) What is the total number of schools in the 8 provinces that took part in 2009 Census@School?

b) The number of schools in Mpumalanga (52) is now added to the total in a).

i) What is the total number of schools in the sample now?

ii) What is the mean number of schools per province now?

c) What does this mean tell us about the number of schools in the sample?

SOLUTION:

total number of schools


a) Mean =
total number of Provinces

total number of schools


66 =
8
* Total number of schools = 8 × 66 = 528

b) Number of provinces with the inclusion of the Mpumalanga = 8 + 1 = 9

i) New total number of schools =528 + 52 = 580

total number of schools 580


ii) New mean = , = = 64.444 ≈ 64
total number of Provinces 9

b) The mean tells us that if the 580 schools were shared out equally amongst the 9 provinces,
each province would get approximately 64 schools.

NB. Order thus no matter in finding the mean.

16
EXAMPLE 5 (GROUPED DATA)
The number of days that the Grade 8 learners were absent at a certain high school during a year was
recorded. This information is represented in the table below.

Time in hours Frequency

1 < y≤ 5 8

5 < y ≤ 10 28

10 < y ≤ 15 27

15 < y ≤ 20 12

20 < y ≤ 25 4

25 < y ≤ 30 1

a) Calculate the mean class for the data.


b) What does this mean tell us about the number of absenteeism in the sample?

SOLUTION:

We fist must find the mid class of EACH class of the time.

Time in hours mi Frequency mi f

1 < y≤ 5 3 8 24

5 < y ≤ 10 7,5 28 210

10 < y ≤ 15 12,5 27 337,5

15 < y ≤ 20 17,5 12 210

20 < y ≤ 25 22,5 4 90

25 < y ≤ 30 27,2 1 27,5

∑ 80 899

17
a) x=
∑ mi × f i = 899 =11,24 hours
∑f 80
b) This mean if the 80 learners were to be absent equally, each learner will be/should absent for
11,24 hours.

CASIO Calculator (fx-82ZA PLUS):

Press Mode than STAT then 1-VAR

Enter all data one at the time pressing =Mi-Values and f after each entry.

Press AC button

Press Shift then STAT then Var

To calculate the mean press 2: x then =

SHARP Calculator (EL-W535SA)

[Press Mode than 1-STAT then 0-SD

Enter all data one at the time in the (x-column) and (f-column) pressing = after each entry.

Press ON/C button

Press Alpha then 8-STAT

then 0-STATISTICS read off the value: x

Both calculators give the value x = 11,2375... ≈ 11,2375 minutes.

18
B) Median:

Arrange the observed values of variable in a data in ascending order (ungrouped data).

1. If the number of observations is odd, then the sample median is the observed value exactly in the
middle of the ordered list.

2. If the number of observations is even, then the sample median is the number halfway between
the two middle observed values in the ordered list.

NB: In both cases, if we let n denote the number of observations in a data set,

then the sample median is at position ( n+12 ) in the ordered list. If a decimal is
found, we add the TWO observed variables and divide their sum by 2.

Example 6

Given the DATA below.

a) 6; 7; 1; 8; 5; 9;17; 11; 23; 10; 4; 5; 15; 12; 3


b) 6; 7;a)
DATA 1; 8; 5; 9;2; 11; 23; 10; 4; 5; 15; 12;3; 17
DATA b)
Determine the Median
First arrange in EACH
the data of the data above.
in ascending First arrange the data in ascending
order: order:
1; 3; 4; 5; 5;6; 7; 8; 9; 10; 11; 12; 1;2; 3; 4; 5; 5; 6; 7; 8; 9; 10; 11; 12;
15;17;23 (n=15 ¿ 15;17;23 (n=16 ¿
There are 15 data items, and 15 is an There are 16 data items, and 16 is
odd number. an even number.

Position is: ( 15+1


2 )
=8 th Position is: ( 16+1
2 )
=8,5 th

The middle item is the 8th item The median is between 8th and 9th
position
1; 3; 4; 5;5; 6; 7;8; 9; 10; 11; 12;
7+8
15;17;23. The Median = 8 Median = =7,5
2

19
C) MODE

20
The mode is the data item that occurs most frequently.

• If there are two modes, then the data set is said to be bimodal’
• If there are more than two modes, then the data set is said to be
multimodal.
• All the data items in a set may be different. In this case it has no
mode.

The associated adjective is modal, so we are sometimes asked to find the modal
value in the ungrouped data and the modal class in the grouped data.

EXAMPLE 7

Find the mode of the following data:

a) 6; 7; 1; 8; 5; 9;5; 11; 2; 10; 4; 5; 15; 12;3; 23

b) 6; 7; 1; 8; 5; 9;5; 11; 23; 10; 4; 5; 15; 12;3; 23

c) 6; 7; 5;1; 8; 9; 11; 3; 10;23;4; 5; 15; 12; 23;10

SOLUTION

c) First arrange the data in ascending a) The data is already arranged in


order: ascending order: 1; 3; 4; 5;5; 6;7;8;
1;2;3;4;5;5;6;7;8;9;10;11;12;15;23 9; 10;10;11; 12; 15;23;23
Look for the value that occurs most There are three modes, so the data
frequently: set is multimodal. Modes = 5; 10 and
1;2;3;4;5;5;6;7;8;9;10;11;12;15;23 23
Mode = 5 b) First arrange the data in ascending
d) The data is already arranged in order: 1; 2; 7; 8; 10; 11; 17; 19; 23;44
ascending order: 1; 3; 4; 5;5; 6;7;8; Look for the value that occurs most
9; 10; 11; 12; 15;23;3 frequently1; 2; 7; 8; 10; 11; 17; 19;
Look for the value that occurs most 23;44
often1; 3; 4; 5;5; 6;7;8; 9; 10; 11;
12; 15;23;23 None of the values are repeated. So,
Advantages and disadvantages of the mean, median and mode
there is No mode
There are two modes; so, the data
is bimodal.
ADVANTAGES DISADVANTAGES
Modes =5 and 23

21
Mean  Easy to work out with a calculator  Can only be used for numbers and
 Uses all the data measurements
 What most people think of as the  Not always one of the values
average  A few very large or small
 Used for quantitative with numbers (outliers) can
symmetric distribution affect its size

Median  Easy to find when the values are in  Can only be used for numbers and
order measurements
 Is one of the values if you have an  A lot of values can take a long time to
odd number of values put in order
 May not be one of the values if you
have an even number of values

Mode  Can be found for any kind of data  Not very useful for small amounts of
 Simple to find because you count, data
not calculate  May be more than one item
 Always one of the items in the data  Does not exist if there is an equal
 should be used when calculating number of each item
measure of centre for the
qualitative variable
 Quick and easy to find from a
frequency table, bar graph or pie
chart.

TYPE OF DATA:

Discrete Data, Continuous Data and Categorical Data

22
.

PERCENTILE

23
Definition: For any number r between 0 and 100, the rth percentile is a
value such that r percent of the observations in the data set fall at or below
that value.

The above figure illustrates the 75th percentile.

 When you write a test and get a mark of 75%, it tells you how many questions you got right. But it doesn’t tell
you how well you did, compared to the other people who wrote the test. Percentiles are values from 0 to 99
that tell you the percentage of the marks that are less than a particular mark.
 If the percentile of your test mark is 75, it tells that
 75% of the marks are LESS than yours.
 100% – 75% = 25% of the marks are MORE than yours

 Percentiles can be used to compare values in any set of ordered data. You can calculate percentiles for
income, mass, etc. Percentiles are often used in education and health-related fields to indicate how one
person compares with others in a group.

 Scores that are in the 95th percentile and above are unusually high while those in the 5th percentile and
below are unusually low.

EXAMPLE 8

 The 89th percentile is a number that 89% of the data items are below.
We then also know that 100% – 89% = 11% of the data items are above that number.
 20th percentile is 12, then 20% of the data items are less than 12, and 80% of the data items more than 12.
 If the 90th percentile is 17, then 90% of the data items are less than 17, and 10% of the data items are more
than 17.

EXAMPLE 9

24
For the following set of 19 data items:

72 71 65 60 62 58 67 57 70 73 50 61 51 55 64 68 69 59 63

a) At which percentile is 70?


b) Find the data item that is at the 20th percentile.
c) What is the median score (the score at the 50th percentile)?

SOLUTION: A stem and leaf diagram can be used to get the data into ascending order:

72 71 65 60 62 58 67 57 70 73 50 61 51 55 64 68 69 59 63

To calculate at which percentile a data item lies:

a ¿ Position=number of participants with less∨equal ¿ 70 ¿ × 100


total number of participants
16
Position= ×100=84,21th
19
Therefor 70 is at 84th percentile

b) To calculate the data item which is at a given percentile i.e. = 20 th percentile.


m
Position: Pm= ×(n+ 1)
100
20
Position: P20 = × 20=4
100
Therefor 4th percentile is 57

c) To calculate the data item which is at a given percentile i.e. 50 th percentile.

m
Position: Pm= ×( n+ 1)
100
50
Position: P50= ×20=10 th
100
Therefor 50th percentile is 63

25
QUARTILES AND THE FIVE NUMBER SUMMARY

a) Quartiles

 Quartiles are the three values Q1, Q2 and Q3 that divide a data set into four approximately equal parts. Each
part consists of approximately 25% of the elements of the data set.

 Q1 is the lower quartile; Q2 is the middle quartile or median and Q3 is the upper quartile.
 The median divides an ordered data set into two halves.
 The quartiles divide an ordered data set into four quarters.
 The median is also the 2nd quartile (M or Q2).

25% of the data 25% of the data 25% of the data 25% of the data

Minimum Lower Median


Upper Maximu
value quartile Q2
quartile m value
Xmin Q1 Q3 Xmax

 From the above diagram, one can see that:


 Approximately one quarter or 25% of the data is less than Q1.
 Approximately three quarters or 75% of the data is more than Q1.
 Approximately one half or 50% of the data is less than Q2 and one half or 50% is more than Q2.
 Approximately three quarters or 75% of the data is less than Q3.
 About one quarter or 25% of the data is more than Q3.
 Approximately one half or 50% of the data lies between Q1 and Q3.
PROCEDURE ON HOW TO FIND THE QUARTILES:
Arrange the observed values of variable in a data in ascending order.

1. If the number of observations is odd, then the sample median is the observed value exactly in the middle of the
ordered list.

2. If the number of observations is even, then the sample median is the number halfway between the two middle
observed values in the ordered list.

NB: If n denote the number of observations in ordered data set, then from the sample:

Q1 is at position ( n+14 ) or 0,25( n+1) in the ordered list.


Q2 is at position 2 ( n+14 ) or 0,5(n+1) in the ordered list.
Q3 is at position 3 ( n+14 ) or 0,75(n+1) in the ordered list.
26
EXAMPLE 10

The scores for 8 golfers who played a single round of


the same golf course are shown below.
Solution:

Order the data:

69, 70, 71, 71, 72, 74, 75, 78 (n = 8)


69 71 75 74 78 70 72 71

Determine the five-number summary Q1 is at position 0,25( n+1)

a. Min value Position= 0,25 ( 8+1 )=2,25th ,


b. Q1
This implies Q1 is between the 2nd and 3rd
c. Q2 position
d. Q3
e. Maxum vale Q1= (70+71)/2 = 70,5
Solution:
Q2 is at position = 0,5(n+1)
Using a calculator:
position = 0,5(8+1)=4,5th
SHARP Calculator (EL-W535SA)
This implies Q2 is between the 4th and 5th
[Press Mode than 1-STAT then 0-SD position

Enter all data one at the time in the Q2 = (71+72)/2 = 71,5

(x-column) pressing = after each entry. Q3 is at position 0,75( n+1) in the

Press ON/C button Position= 0,75 ( 8+1 )=6,75th

Press Alpha then 8-STAT This implies Q3 is between the 6th and 7th
position
then 0-STATISTICS read off the values,
Q3 = (74+75)/2 = 74,5
using the scroll button.
xmin = 69 and xmax = 78

NB: If n denote the number of observations in ordered data set, then from the sample:

Q1 is at position ( n+14 ) or 0,25(n+1) in the ordered list.


Q2 is at position 2 ( n+14 ) or 0,5( n+1) in the ordered list.

27
Construction of Box-and-whisker

1. Draw a horizontal (or vertical) measurement scale.

2. Construct a rectangular box with a left (or lower) edge at the lower quartile and a right (or

upper) edge at the upper quartile. The box width is then equal to the iqr.

3. Draw a vertical (or horizontal) line segment inside the box at the location of the median.

4. Extend horizontal (or vertical) line segments, called whiskers, from each end of the box to the smallest and
largest observations in the data set.

EXAMPLE 11

Draw a box-and-whisker diagram of the following set of data:

506 503 507 504 510 511 526 513 517 508 515 513 508 509 516

SOLUTION:

STEP 1: Arrange the data in ascending order

503 504 506 507 508 508 509 510 511 513 513 515 516 517 526 . You may use the stem and leaf.

STEP 2: Find the five-number summary.

503 504 506 507 508 508 509 510 511 513 513 515 516 517 526

Minimum value = 503 Q1 = 507 M = Q2 = 510 Q3 = 515

Maximum value = 526

STEP 3: Draw a number line long enough to go from 503 to 526.

STEP 4: Draw vertical lines at Q1, M and Q3 and form the box

STEP 5: Join the box to the minimum and maximum values to form the whiskers.

28
Interpretation: We can then say the data is skewed to the right/ positively skewed, since the right side of the Box
is more than the left. This implies that the upper half of the data is more spread than the lower half. NB the mean
is >510

SYMMETRIC AND SKEWED DATA

 A measure of shape describes the distribution of the data within a data set.
 A distribution of data values can be symmetric or skewed.
 In a symmetric distribution, the two sides of the distribution are a mirror image of each other
 In a skewed distribution, the two sides of the distribution are NOT mirror images of each other
 Both frequency polygons and box-and-whisker diagrams can be used to illustrate symmetric and skewed
data.

Mean
Mode Mode
Mode Median
Median Median
Mea Mean
n

Long-Tail
Long-Tail

Left-Skewed (negative Skewness) Symmetric Right-Skewed (Positive Skewness)

Left-Skewed (negative Skewness)

Symmetric

Right-Skewed (Positive Skewness)

29
KEY FEATURES OF A SYMMETRIC DISTRIBUTION

• The shape is symmetrical

• The mode, median and mode have the same value. ( mean−median=0 ¿

• Most of the data are clustered around the centre.

KEY FEATURES OF SKEWED DATA

 Skewness is the tendency for the values to be more frequently around the high or low ends of the x-axis.
 With a positively skewed distribution, the tail on the right side is longer than the left side.
( mean−median>0)
 With a negatively skewed distribution, the tail on the left side is longer than the right side.
( mean−median<0)

OUTLIERS

 An outlier is a data entry that is far removed from the other entries in the data set e.g. a data entry that is
much smaller or much larger than the rest of the data values.
 Any data item that is less than Q1 – 1,5 × IQR OR more than Q3 +1,5 × IQR is an outlier

NOTE:

An outlier has an influence on the mean and the range of the data set,
but has no influence on the median or lower or upper quartiles
An outlier can affect the skewness of the data.

MEASURES OF DISPERSION

 The measure of dispersion helps us to study the variability of the items. In a statistical sense, dispersion

30
has two meanings:

The data can either:


 Be grouped closely together around the measure of central tendency, or
 Be spread widely apart around the measure of central tendency.

a) RANGE

 The range is the simplest measure of spread. It is the difference between the largest and smallest
items of data.
 Range = Largest Value – Smallest Value
 There are some limitations to using range:
 It does not consider anything about the distribution of any other piece of data except the
smallest and largest value.

b) INTERQUARTILE RANGE
 The interquartile range (or IQR) is the difference between the upper quartile and the lower
quartile. Interquartile Range = Q3 – Q1
 The interquartile range is a better measure of dispersion than the range. It is not affected by any
extreme values (very small or very large values). It is based on the middle half of the data.
It is the range between the upper and lower quartiles.
 The semi-interquartile range is sometimes used. It is half of the interquartile range.
Q3−Q1
Semi-interquartile range =
2

b) Standard deviation and Variance.


 The variance is the mean of the sums of the squares of the deviations from the mean.

o Variance:( σ x ) =∑ ¿ ¿¿ ¿
2

 When data elements are tightly clustered together, the standard deviation and variance are small;
when they are spread apart, the standard deviation and the variance are relatively large.
 A data set with more data near the mean will have less spread and a smaller standard deviation
 A data set with lots of data far from the mean which will have a greater spread and a larger
standard deviation.
 Standard deviation (SD) is the most used measure of dispersion.

31
Example 12 Solution

Fifteen members of a soccer team took a part in a a) position = 0,5(15+ 1)= 8th
tournament. Each player was allowed the same amount of
time on the soccer field. The points scored by each player M = Q2 = 42
at the end of the tournament are as shown below.
b) Range=x max −x min
27 28 30 32 34
Range=62−27=35
38 41 42 43 43
c)position = 0,24 (15+1)=4th
44 46 53 56 62
Q1 = 32
Determine the following:
position = 0,75(15+ 1)=12th
a) Median of the given data
Q3 = 46
b) Range
IQR¿ Q 3 – Q1=46−32=14
b) Interquartile range
c) x=41,27
c) mean
d) σ x =√ ∑ ¿ ¿ ¿ ¿ ¿
d) standard deviation

Using a calculator: σ x=
√ 1476,9333
15

σ x =9,92

EXAMPLE 13

32
The box and whisker diagrams below represent Vuyo and Fela scores for their School Based Assessment Task in a
certain subject throughout the year.
𝐈𝐧𝐭𝐞𝐫𝐪𝐮𝐚𝐫𝐭𝐢𝐥𝐞 𝐑𝐚𝐧𝐠𝐞 = Q3 – Q1 = 16,8 − 13,9 = 2,9

Vuyo’s scores

Fela’s scores

68 70 72 74 76 78 80 82 84 86 88 90 92 94 96

a) Give the range of Fela’s scores.


b) Give the minimum of Vuyo’s scores
c)Comment on who think had a more consistent performance throughout the year. Motivate your answer by
referring to the values of the box-and whisker diagram.
SOLUTION:
a) Range= 96-68 = 28
b) xmin = 76
c) Vuyo is more consistent during the year, the range and the IQR of Vuyo scores are smaller than Fela’s
Activity 1
1. The following times (in seconds) for a 100 m race were recorded at an athletics meeting

10,5 13,9 13,9 14,7 14,7 15,1 15,3 15,9 17,7 18,5

1.1. Calculate the five-number summary for the data set. (4)
1.2. What was the winner’s time for the race? (1)
1.3. Calculate the 80th percentile of the data set. (2)
1.4. Calculate the inter-quartile range of the data set. (1)

Activity 2
2.1 Twenty learners were asked in a survey, the number of hours in a week they spent. Texting on their cell phones.
Their answers, correct to the nearest hour, have been ordered into a list as follows:

3 5 6 6 7 8 8 9 9 11 12 13 14 15 15 18 22 24 25 30

2.2.1. Give the five-number summary of the data set. (4)


2.2.2. Draw a box-and-whisker plot for this data set. (3)
2.2.3. Calculate the inter-quartile range of the data. (1)
2.2.4. Describe the distribution of the data. (1)

Activity 3

33
Nineteen Girls were required to complete a puzzle as quickly as possible. There times (in seconds) were recorded and
are as shown below.
14 15 16 16 17 17 17 18 18 19 19 19 20 21 22 23 24 24 29
3.1. Identify the median time taken by the girls to complete the puzzle . (1)
3.2. Determine the lower and the upper quartiles of the data. (2)
3.3. Draw the box and whisker diagram to represent the data. (2)
3.4. the five-number summary of the time (in second) taken for 19 boys to complete the same puzzle is
(15; 19; 23; 26; 30).
3.4.1 Calculate the interquartile range for the time taken for the boys. (2)
3.4.2 If only one boy took 19 seconds to complete the puzzle, what percentage of the boys took at least 19 seconds to
complete the puzzle? (1)
3.6 In which group Girls or boys, did a large number of learners complete the puzzle in less than 23 seconds? Justify
your answer. (2)

Activity 4
A machine is supposed to cut larger blocks of butter into 500g blocks.
Thirteen 500g blocks of butter were weighed and the results were recorded
below.

50 498 499 501 50 495 510 520 51 502 491 502 503
6 3 1

4.1. List the five-number summary of the data set. (4)

4.2. Calculate the inter-quartile range for the data set. (1)

4.3. Draw a box-and-whisker plot for this data set. (3)

4.4. Describe the distribution of the data. (1)

Activity 5
The height of 20 children were measured (in centimetres) and the results were recorded. The data collected is given
below.
127 128 129 130 131 133 134 134 135 136

34
137 138 139 140 141 142 142 143 144 145

5.1 Write down the median height measured. (1)


5.2 Determine
5.2.1 The mean height. (2)
5.2.2. The range. (1)
5.2.2. Interquartile range. (3)
5.3 Draw the box and whisker diagram to represent the data. (2)
[9]
Activity 6
6.1. The ordered test results of a class of 15 learners are shown below.
The test was out of 10 marks.

2 2 3 3 4 4 4 5 5 5 7 7 8 9 1
0
6.1. Calculate the first, second and third quartiles
for the data set. (3)
6.2. Calculate the 40th percentile of the data set. (2)
6.3. Draw a box-and-whisker plot for this data set. (3)
6.4. Calculate the interquartile range of the data set. (1)
6.5. Calculate the semi-interquartile range of the data set. (1)
6.6. What does the semi-interquartile range tell us about the distribution of the middle 50% of this data set?
(1)
Activity 7
The individual ages of the employees of an IT company are as follows:

21 27 23 25 29 18 53 34 28 36 29 36 37 29 29 35 43 53 47 31
7.1 Calculate the mean of the employees’ ages.
7.2 Calculate the standard deviation of their ages.
7.3 How many of the employees’ ages are within ONE standard deviation of the
mean?
7.4 Draw a box-and-whisker diagram of the data using the answer sheet.
7.5 Comment on the spread of the ages of the workers.

Activity 8 31 36 62 74 66 37 46 40 52 56 63
A baker keeps records
60 of the number
34 46 of scones
39 he 43sells each
65 day.The
31 data48
for the 19 days is shown below:

35
8.1 For the given data determine:
8.1.1 the mean. (2)
8.1.2 the median. (2)
8.2 Draw the box and whisker diagram to represent the data. (2)

Activity 9
The box and whisker diagrams of two sets A and B are shown below

4 8
12
2 14 Data A

2 18 Data B

4 6 10

9.1 Write down what is common to both sets of data. (2)


9.2 Which data set is symmetric? State the reason. (2)
9.3 Is the other data skewed to the left, or right? State the reasons. (2)
Activity 10.
The data below shows the number of laptops sold by 15 sales agents during the last financial year
43 48 62 52 46 90 58 37
48 73 84 68 54 34 78
10.1 Determine the median of the number of laptops sold.
10.2 calculate the mean of the data
10.3 Calculate the interquartile range IQR
10.4 Draw the box and whisker diagram for the data above
10.5 Calculate the variance of the data
10.6. Identify the outliers
Activity 11 7 8
8 4 5 8 8
11. A group of 30 learners had to show with their
9 2 3 4 6 7 7 8 9
hands, how long they thought a metre was.
10 0 1 2 3 3 3 5 6 7 9
Their estimates were measured and
11 3 5 5 6
12 5 8 36
13 7
summarised in
the stem-and-leaf diagram alongside.

11.1. Calculate the five-number summary for the


data set.
11.2. Noma estimated the length of one metre to be
93 cm. Is her estimate above or below the 20 th percentile?
11.3. Draw a box-and-whisker plot for this data set.

Activity 12
In the table below a, b, c, d, e, f, and g represent values in the data set written in an increasing order. No value in
the data set is repeated.

a b c d e f g

Determine the values of a, b, c, d, e, f, and g if


o the maximum is 42
o the range is 35
o the median is 23
o the difference between the median and upper quartile is 14
o The interquartile range is 22
o e =2c and mean is 25

UNIT 2: Grouped data

INTRODUCTION

37
In this unit participants will look at the concept of converting ungrouped data to grouped data,
Measures of central tendency, five number summaries, measure of dispersion (Quartiles) and
different graphs in statistics

LEARNING OUTCOME

• Draw and interpret a histogram

• Draw frequency polygons

• Draw and interpret ogives (cumulative frequency curves)

• Determine whether data is symmetric or skewed

• Identify the values of the outliers.

STATISTICAL GRAPHS

Organised data can often be presented in graphical form.

• Statistical graphs are used to describe data or to analyse it.

• The purpose of graphs in statistics is to communicate the data to the viewers in pictorial form. It
is easier for most people to understand data when it is presented as a graph than when it is
presented numerically in tables.

In earlier grades you dealt with the following graphs

Types of graphs

Broken-line graph PI- Graph

OrganisingBar
thegraph
ungrouped data in a grouped frequency tableHistogram

38
The main steps in a process of grouping quantitative variable into classes:

STEP 1: Find the minimum and the maximum values the variables have in the

data set

STEP 2: Choose intervals of equal length that cover the range between the minimum

and the maximum without overlapping. These are called class

intervals, and their end points are called class limits.

STEP 3: Count the number of observations in the data that belongs to each class

interval. The count in each class is the class frequency.

STEP 4: Calculate the total frequencies for each which should be equal to the total number of observations in the
data.

The data in the table below represents the MATHEMATICS test marks of the learners out of 50 marks.

Complete the table below:

Activity A:

Data 1 Ungrouped Data 2 grouped

26 8 17 16 28 Class frequency

32 25 14 45 41 0-10

39 49 33 22 27 11-20
22 6 22 36 46
21-30
25 18 21 6 11
31-40

41-50

HISTOGRAMS

A histogram gives us a visual interpretation of data. It looks very similar to a bar graph, but there are

39
definite differences between them.
HISTOGRAM BAR GRAPH
• It is a representation of grouped data • It is a representation of ungrouped
data that does not have to be
• There is no gap between the bars
numerical
For example, you draw a HISTOGRAM to show the
• There is generally a gap between
number of people whose heights (h) lie in the
the bars
following intervals (measured in cm): 150 ≤ h <
160. For example, you draw a BAR GRAPH
to show the number of learners in a
160 ≤ h < 170; etc.
class who wear glasses and the
number who do not wear glasses

EXAMPLE 14

The individual masses of (in kg) of 25 rugby players are given below:
75 76 60 75 90 81 93 88 102 78
68 90 80 77 81 69 60 83 91 100
80 70 81 64 80
Complete the table:
Percentages Frequency
60 ≤ m < 70
70 ≤ m < 80
80 ≤ m < 90
90 ≤ m < 100
100 ≤ m < 110
TOTAL
a) Organise the data using a grouped frequency table.
b) Draw a HISTOGRAM to illustrate the data.
c) Calculate the modal interval. What does this measure of central tendency tell you about the Rugby
players mass?
d) Estimate the median. What does this measure of central tendency tell you about the Rugby players
mass?

SOLUTION:

a) Draw the histogram as follows:

Percentages Frequency

40
60 ≤ m < 70 5

70 ≤ m < 80 6

80 ≤ m < 90 8

90 ≤ m < 100 4

100 ≤ m < 110 2

TOTAL 25

b)

STEP 1: Draw and label the horizontal and vertical axes.

STEP 2: Represent the frequency on the vertical axis and the classes on the horizontal axis.

STEP 3: Using the frequencies (or number of Rugby players) as the heights, draw vertical bars for each

class.

Mass of Rugby players(HISTOGRAM)


Number of Rugby Player

10
8
6
4
2
0
60-70 70-80 80-90 90-100 100-110
Mass (in kg)

c) The modal interval is the interval with the largest frequency or largest number of learners. So, the modal
class is the interval is 80 ≤ m < 90. This tells us that more rugby players mass is in the interval 80 ≤ m< 90
than in any of the other intervals.

d) There are 25 data items (masses). The median lies between the 12th and the 13th mass. Add up the
frequencies until you reach 12 (or more than 12): 5 + 6 + 8 = 19 The 19th mass lies in the interval 80 ≤ m < 90 So
the median lies in the interval 80 ≤ m < 90 The median ≈ 85kg (the midpoint of the interval) This tells us that 50%
of the rugby players weigh less than 85 kg and 50% of rugby players weigh more than 85kg

NOTE:

A histogram should have the following:

• A title which describes the information that is contained in the histogram.

• A horizontal axis with a label which shows the scale of values into which the data fit (grouped 41
data intervals)

• A vertical axis with a label which shows the number of times the data within the interval
occurred (frequency)
Activity 1

1. The frequency table below represent the distribution of the amount of time (in hours) that 80 high school
learners spent in one week watching their favourite sport.

Time in hours Frequency

10 < t ≤ 15 8

15 < t ≤ 20 28

20 < t ≤ 25 27

25 < t ≤ 30 12

30 < t ≤ 35 4

35 < t ≤ 40 1

1.1 Draw a HISTOGRAM to represent the data.


1.2 Calculate:
1.2.1 the modal interval
1.2.2 an estimate of the median
1.3 What do these two measures of central tendency tell you about the amount of time the learners devote to
watching their favourite sport

Learners Watching TV
30
Numbers of Lernars watching TV

25

20

15

10

0
10 -15 15 - 20 20 - 25 25- 30 30 -35 35-40
Time (hours)

ACTIVITY 2:

In the 2009 Census@School, learners were asked which their favourite subjects at school were. Fifty Grade 11

42
learners from a certain school in Limpopo chose Science as their favourite subject. The following are their Science
marks (as percentages):

31 62 51 44 61 63 59 47 59 67

50 54 61 41 48 74 53 53 53 36

60 42 50 48 42 27 43 42 43 54

49 47 51 28 54 48 83 65 54 35

61 56 57 32 38 32 40 63 56 59

2.1Organise the data in a grouped frequency table.

2.2 Draw a HISTOGRAM to represents the data.

2.3 Calculate the modal interval and an estimate of the median and say what these two measures of central

tendency tell you about the learners’ mark.

Activity 3

The number of days that the Grade 8 learners were absent at a certain high school during a year
was recorded. This information is represented in the table below.

Time in hours Frequency

1 < y≤ 5 8

5 < y ≤ 10 28

10 < y ≤ 15 27

15 < y ≤ 20 12

20 < y ≤ 25 4

25 < y ≤ 30 1

43
1)Write down the modal class for the data.

2)How many learners were absent from school for less than 15 days?

3)How many Grade 8 learners are at the school?

4)Draw a HISTOGRAM to represent the data above.

5)Use the information on the table/graph to determine the median number of days the Grade 8
learners were absent

FREQUENCY POLYGONS

To draw the frequency polygon:

 A frequency polygon can be used instead of a histogram for illustrating grouped data.
 One way of drawing a frequency polygon is to

a) Draw a histogram rugby players Mass

b) Join the midpoints of the top of the columns of the histogram

c) Extend the line to the midpoint of the class interval below the lowest value and to the midpoint of the class
interval above the highest value so that the line touches the horizontal axis on both sides.

44
 Another way of drawing a frequency polygon is to

a) Calculate the midpoint of each interval and then to plot the ordered pair (midpoint of the interval; frequency)

b) Plot the midpoint of the interval below the lowest interval and the interval above the highest interval and plot
the points (midpoint of the interval; 0)

c) Join these points with straight lines.

EXAMPLE 15

The Grade 10 and Grade 11 learners were surveyed to find out the approximate number of hours every week they

45
spend doing their Mathematics and Science homework. The results are summarised in the following grouped
frequency table:

Number of hours spent on Number of Number of


Mathematics and Science homework Grade 10 Grade 11
each week learners learners

5 ≤ t < 10 3 12

10 ≤ t < 15 4 22

15 ≤ t < 20 7 10

20 ≤ t < 25 19 6

25 ≤ t < 30 16 8

30 ≤ t < 35 1 2

TOTAL 50 60

15.1 Draw two frequency polygons on the same set of axes to illustrate this data.

Use the table and the graphs to answer the following:

15.2 What is the modal interval for Grade 10 and for Grade 11 respectively?
15.3 Approximately how many more Grade 11 learners than Grade 10 learners spent between 15 and 20 hours
doing their homework each week?
15.4 Which grade spent more time doing their homework?

SOLUTION

Number of hours spent on Mid-point of the Number of Grade Number of Grade


Mathematics and Science interval 10 learners 11 learners
homework each week

0≤t<5 2,5 0 0
5 ≤ t < 10 7,5 4 12
10 ≤ t < 15 12,5 4 22
15 ≤ t < 20 17,5 7 10
20 ≤ t < 25 22,5 19 6
25 ≤ t < 30 27,5 16 8
30 ≤ t < 35 32,5 1 2
35 ≤ t < 40 37,5 0 0
Frequency Polygon

15.1

46
15.2 The modal interval for Grade 10 is 20 < t ≤ 25

The modal interval for Grade 11 is 10 < t ≤ 15

15.3 Difference in the number of learners who spent between 15 and 20 hours doing

homework each week = Number in Grade 11 – Number in Grade 10 = 10 – 7 =3

So, 3 more Grade 11 learners than Grade 10 learners spent between 15- and 20-hours doing homework each week

15.4 According to the table:

36 out of 50 Grade 10 learners (72% of them) spent 20 hours or more doing homework each week.

16 out of 60 Grade 11 learners (27% of them) spent 20 hours or more doing homework each week.

So the Grade 10s spent more time on homework than the Grade 11s.

Activity A.

1. The frequency table below represent the distribution of the amount of time (in hours) that 80 high school
learners spent in one week watching their favourite sport.

Time in hours Frequency

10 < t ≤ 15 8

15 < t ≤ 20 28

20 < t ≤ 25 27

25 < t ≤ 30 12

30 < t ≤ 35 4

35 t ≤ 40 1

47
1.1 Draw a FREQUENCY POLYGON to represent the data.
1.2 Calculate:
1.2.1 the modal interval

1.2.2 an estimate of the median

1.3 What do these two measures of central tendency tell you about the amount of time the learners devote
to watching their favourite sport

OGIVES / CUMULATIVE FREQUENCY CURVES

CUMULATIVE FREQUENCY

Cumulative frequency shows the number of results that are less than (<) or less than or equal to (≤) a stated
value in a set of data.

To find the cumulative frequency,

• Add up the frequencies as you go down the frequency table.

• Write each running total or cumulative frequency in your table

48
Frequency tells us how many of each item there are in a data set.

For example

As part of the Census@School, 170 learners were surveyed to find out the type of dwelling that they lived in.

The following table shows the result of the survey:

Type of house that you live in Frequency (number of learners)

Traditional dwelling 7

House on separate yard 76

Tent 1

Informal dwelling in an informal settlement 86

TOTAL 170

For example

Using the above information, we can find the cumulative frequency

Type of house that you live in Frequency Cumulative Frequency

(number of learners)

Traditional dwelling 7 7

House on separate yard 76 7 + 76 = 83

Tent 1 83 + 1 = 84

Informal dwelling in an informal settlement 86 84 + 86 = 170

TOTAL 170

 Can you see that the last cumulative frequency is equal to the total frequency?
 You can find cumulative frequencies of discrete data and continuous data

49
 An ogive or cumulative frequency curve is a graph that shows the information in a cumulative frequency
table. The graph is useful for estimating the median and inter-quartile range of the grouped data.
 You can draw an ogive of ungrouped discrete data, grouped discrete data or grouped continuous data. It
can be drawn from a grouped frequency table or an ungrouped frequency table.

Example 16.

The following frequency table shows the time (in minutes) taken by learners to travel to school.

Time taken to travel Frequency Cumulative Ordered Pairs


to school Frequency

0 < t ≤ 10 4

10 < t ≤ 20 12

20 < t ≤ 30 28

30 < t ≤ 40 32

40 < t ≤ 50 29

50 < t ≤ 60 15

16.1 Complete the table.


16.2 Draw an ogive to illustrate the information

SOLUTION:

16.1

a) Steps to follow when completing the table:


b) • Add in an interval with a frequency of 0 before the first interval.
c) • Find the cumulative frequency by adding the frequencies.
d) • List the ordered pairs where the first coordinate = upper limit of the interval and the second coordinate
= cumulative frequency.

Note: A cumulative frequency of 105 means that 105 learners or less spent 50 minutes or less to walk to school.

Time taken to travel Frequency Cumulative Ordered Pairs


to school Frequency

-10 < t ≤ 0 0 0 (0;0)

0 < t ≤ 10 4 4 (10;4)

10 < t ≤ 20 12 16 (20;16)

20 < t ≤ 30 28 44 (30;44)

50
30 < t ≤ 40 32 76 (40;76)

40 < t ≤ 50 29 105 (50;105)

50 < t ≤ 60 15 120 (60;120)

16.2 Drawing the ogive as follows:

i) Draw the axes and label the variable on the x-axis and the cumulative frequency on the y-axis.
ii) Plot the ordered pairs.
iii) Join the points to form a smooth curve.

Ogive Showing the time to


travel to school
140

120

100
Cumulative frequency

80

60

40

20

0
-5 5 15 25 35 45 55 65
Time taken to travel(min)

 Always remember when drawing cumulative frequency curve from a table of grouped data, the
cumulative frequencies are plotted at the upper limit of the interval.

Interpretations:
From the OGIVE the 5-Number summery could be calculated or determined via interpolation.

51
Activity 1

In 20009 Census@School leaners were asked what their arm span was, correct to the nearest
centimetres. The results of the two hundred of grade 10, 11 and 12 learners who took part were
recorded as follows:

Time taken to travel Frequency Cumulative Ordered


to school Frequency Pairs

130< h ≤ 135 16

135< h ≤ 140 26

140 <h ≤ 145 42

145<h ≤ 150 54

150 <h ≤ 155 26

155 <h ≤160 22

160 <h ≤165 14

1.1 Copy and complete the table

1.2 Draw an ogive to illustrate the data

1.3 Use the ogive to determine the approximately how many learners have arm

span that are less than or equal to 152 cm.

1.4 Use you graph to determine approximately how many learners have arm

spans between 138 and 158 cm?

52
Activity 2

Fifty learners who travels by car to school were asked to be recorded the number of kilometres travelled to and
from school in one week. The following table shows the results:

Time taken to travel Frequency Cumulative Ordered


to school Frequency Pairs

10< x ≤ 20 2 2

20< x ≤ 30 9

30 <x ≤ 40 13

40<x ≤ 50 26

50 <x≤ 60 32

60 <x≤ 70 50

2.1 Copy and complete the table

2.2 Draw an ogive to illustrate the data

2.3 Use the ogive to estimate the median number of kilometres travelled per week.

Activity 3

The masses of a random sample of 50 boys in grade 12 were recoded. The cumulative frequency graph
(ogive)represents the recorded masses.

53
Ogive Showing the Masses of Boys
60
Cumulative frequency

50
40
30
20
10
0
50 60 70 80 90 100 110
Mass of Boys(kg)

3.1 How many of the boys had a mass between 90 and 100 kilograms?
3.2 Estimate the median of the boys.
3.3 Estimate how many boys had a mass less than 80 kilograms.

Activity 4

The arm spans (in cm) of the eleven players in each of the two different soccer TEAMS A and B are recorded.

The arm span for TEAM A are:

203,214,187, 196, 205, 203, 199, 199,194 and 206.

4.1 Calculate the mean of the arm spans.

4.2 Complete the table below.

TEAM A arm span x i−x ( x i−x 2 )


(cm) x i

203

214

187

196

205

203

199

54
199

194

206

n=¿ ∑ ( x i−x 2)=¿

4.3 Calculate the standard deviation of the arm span using the data on the table.

4.4 For TEAM B the variance is 875 cm 2 calculate the standard deviation

for TEAM B.

4.5 Make a comment on the dispersion of both teams, of the arm span team players

on both teams.

Activity 5

For 2009 at a certain school, 47 Grade 12 learners recorded how long (in cm,) it took them to travel to school. The
following data was recoded.

Time (in min) Frequency

5< x ≤ 10 1

10< x ≤ 15 5

15 <x ≤ 20 9

20<x ≤ 25 `3

25 <x≤ 30 11

30 <x≤ 35 8

5.1 Use the information to determine the five-number-summary

5.2 Draw a box-and-whisker diagram to illustrate the five-number summery

5.3 Comment on the spread of the time taken to complete the task.

Activity 6

Three high schoola in Limpopo have a total of 132 Grade12 learners. These learners completed a question in 2009
Census@Schoool were they were asked to record the distance(in km)they travel each day form home to school.
The results of the survey are as shon in the grouped frequency below.

Distance (in km) Frequency Midpoint of the interval

55
0< x ≤ 5 12

5< x ≤ 10 29

10 <x ≤ 15 13

15<x ≤ 20 63

20 <x≤ 25 12

25 <x≤ 30 3

6.1 Copy and complete the table

6.2 Draw a frequency polygon to illustrate the data.

6.3 Determine the median of the data in the table.

6.4 Use your calculator to determine the mean of the data

6.5 Calculate the D=mean−median

6.6 By refering to the shape of the polygon and the relationship between the mean and median, state whether
distribbution of data is SYMETRIC, POSITIVELY SKEWD or NEGATIVELY SKEWED.

Activity 7

A group of learners were interviewed to find out how much money they spend on cell phone airtime during
the week. The outcomes were recorded in the table below:

Rands Frequency Cumulative Ordered


Frequency Pairs

20< x ≤ 40 16 16

40< x ≤ 60 44 A

60 <x ≤ 80 x 116

80<x ≤ 100 y C

100 <x≤ 120 10 156

7.1 Calculate the numerical values of a, x ,y and c

7.2 What percentage of learners spend less than R50 on airtime

7.3 Draw an ogive to represent this data set.

7.4 Use the graph to predict how many learners used less than R50 on airtime per week.

7.5 Use the ogive to determine the median. Clearly indicate where the reading is taken.

Activity 8

56
Simon collected cans for recycling fo 21 days. The numner of cans collected by Simon was recorded
and the data was given below.

79 77 80 80 71 67 51

68 70 76 79 82 82 50

84 73 52 80 63 73 78

8.1 Draw a box-and whisker diagram to represent the data

8.2 Determine the mean of the data

8.3 Calculate the standard deviation of the data

8.4 For how many days did Simon collect the cans that within ONE standard deviation.

8.5 Are there any outliers in the data if yes, substantiate you answer.

Activity 9

A group of 50 Gr 11 learners participated in a cross-country race. The times (in minutes) taken by the 50 learners
to complete the course are given in the table below.

Time (in minutes) Frequency Cumulative frequency

15 ≤t <19 6

19 ≤t <23 11

23 ≤ t <27 15

27 ≤ t<31 12

31 ≤t <35 4

35 ≤ t<39 2

Use the answer sheet provided and:

9.1 Complete the cumulative frequency column. (2)

9.2 Draw an ogive for the given data. (4)

9.3 Determine from the ogive the:

9.3.1 median. (1)

9.3.2 lower quartile. (1)

57
9.3.3 upper quartile. (1)

Activity 10

The number of days that the Grade 8 learners were absent at a certain high school during a year
was recorded. This information is represented in the table below.

Time in hours Frequency

1 < y≤ 5 8

5 < y ≤ 10 28

10 < y ≤ 15 27

15 < y ≤ 20 12

20 < y ≤ 25 4

25 < y ≤ 30 1

1)Write down the modal class for the data.

2)How many learners were absent from school for less than 15 days?

3)How many Grade 8 learners are at the school?

4)Draw a cumulative frequency graph (OGIVE) to represent the data above.

58
5)Use the cumulative frequency graph to determine the median number of days the Grade 8
learners were absent

Ogive Number of leaners-hours absent


90
80
70
Number of Learners

60
50
40
30
20
10
0
0 5 10 15 20 25 30 35
Hours Absent

UNIT 3: Univariate and bivariate data

INTRODUCTION

In this unit participants will look at the concept


 Univariate and bivariate data
LEARNING OUTCOME

At the end of this Unit, participants should be able to:


• distinguish between univariate and bivariate data.
• Draw scatter plots.
• Describe the correlation between any two sets of data in a scatter plot.
• Determine whether the correlation is linear, quadratic or exponential.
• Calculate the correlation coefficient.
• Draw an intuitive line of best fit
• Find the equation of regression lines.
• Use regression lines to make predictions

UNIVARIATE AND BIVARIATE DATA

59
• UnivariateDATA
BIVARIATE means “one variable” (one type of data).
Bivariate datadata
• Univariate involves twoavariables,
involves and you investigate these two variables to find out if there are
single variable.
connections between sets of data.
• univariate data can be summarised using a pictograph, bar graph, pie chart, histogram, frequency
For example,
polygon, line ifgraph,
you investigate
broken lineheight
graphand mass, you probably will find that taller learners have a
or ogive.
greater mass than shorter learners.
Examples
• You can graph bivariate data on a scatter plot (also called a scatter diagram or scatter graph).
 The time taken by learners to get to finish writing an assignment
Examples of bivariate
 Accounting data
test are:
marks.
 shoe size and the length of a person’s forearm,
 time spent watching television and mathematics test scores,
 hand span and foot length,
 the speed of a car and the petrol consumption.
 the age of a learner and the grade she is in.

* A scatter plot is a graph


that helps us
to see whether there is a
correlation
(relationship) between any
set of two
numeric data.

• It has two axes, one for


each variable.

• Each ordered pair of • In this scatter plot the points are spread
values is plotted as a point going up from left to right. This shows a
on the graph. relationship between the two variables.

The x-coordinate is the • The point O (18; 5) is an outlier. An outlier


independent can have an extreme x-value, an
extreme y-value, or both.
variable.

The y-coordinate is called It means that an older child is in a lower


the grade.

dependent variable.

60
• An independent variable is the variable that determines the value of another variable
(the dependent variable). This variable can often be manipulated.

• A dependent variable is the variable whose values depend on the

corresponding values of the domain or independent variable.

Types of Correlation

In analysing the scatter plot, you look for a pattern in the way the points lie.

Certain patterns tell you that correlations (relationships) exist between the two

variables.

When describing the relationship between two variables displayed on a scatter

plot, we should comment on:

a) The form – whether it is linear or non-linear (either a quadratic or

exponential curve).

b) The direction – whether it is positive or negative

c) The strength – whether it is strong, moderate or weak.

Table showing different types of correlation

The points are scattered randomly over the


graph indicating no pattern between the
two sets of data. There is no correlation
between the two variables.

As one variable increases, the

other variable also increases. Such a


pattern shows a strong positive
correlation.

61
As one variable increases, the

other variable decreases. Such a pattern


shows a strong negative correlation.

The points are obviously clustered from


bottom left to top right but are not
clustered together as closely as with the
strong positive correlation. There is a
moderately positive correlation.

The points are obviously clustered from top


left to bottom right, but are not clustered
together as closely as

with the strong negative correlation.

There is a moderately negative correlation.

ACTIVITY

1 Describe the relationship that exists between the variables in the scatter plots
below.

Say whether there is:

• Zero correlation

• A strong positive correlation

• A strong negative correlation

• A moderate positive correlation

• A moderate negative correlation

• A non-linear correlation (either a quadratic or exponential relationship).

62
2 The scatter graph below shows the shoe sizes and heights of a group of 9 girls.

a) What shoe size does the tallest girl wear?

63
b) How tall is the girl with the largest shoe size? Give your answer in metres.

c) Does the shortest girl wear the smallest shoes?

d) What do you notice about the shoe sizes of the taller girls compared to the
shoe size of the shorter girls?

e) Describe the correlation shown by this scatter graph

CORRELATION COEFFICIENT

A commonly used statistical measure of association is called the correlation


coefficient ,(r).

The correlation coefficient is a measure of the strength and direction of the


linear relationship between two variables.

The range of the correlation coefficient is –1 to 1.

• If x and y have strong positive correlation, r is close to 1.

• If x and y have strong negative linear correlation, r is close to –1.

• If there is no linear correlation or there is a weak linear correlation, r is close

to 0.

The correlation coefficient ,r, has no units.

The correlation coefficient only measures the strength of a linear association.

64
65
Calculation of the correlation coefficient r using a calculator

Example

In the 2010 Household Survey, a representative sample of people was

asked how many rooms they have in their homes. The table below

shows the data taken from the Gauteng Province.

Number of
1 2 3 4 5 6 7 8 9 10
rooms

Percentage of
10,6 13,9 10,9 17,9 13,1 9,8 6,1 3,8 2,6 1,3
households

Use your calculator to find the correlation coefficient r

Demonstration

Solution: CASIO Calculator (fx-82ZA PLUS):

Using a calculator: [MODE] [2 : STAT] [1: 2– VAR]

SHARP Calculator (EL-W535SA) Enter all data one at the time in the

[Press Mode than 1-STAT then 1- a + bx (x-column) pressing = after each entry.

Then: 1- a + bx
Enter all data one at the time in the
Enter all data one at the time in the
(x-column) pressing = after each entry.
(y-column) pressing = after each entry
Then:
Enter all data one at the time in the
Enter all data one at the time in the
(x-column) pressing = after each entry.
(y-column) pressing = after each entry
[AC] then SHIFT 1-STAT then 5- Reg] then 3-r
Press ON/C button followed by =

Press Alpha then 8-STAT Display r = -0,92

then 1-REGRESSION read off the values,

using the scroll button.


a = 20,9; b = -1,98; r = -0.92 Display

66
Activity

LINE OF BEST FIT

If there is a linear correlation between the two sets of the data, it may be possible to
draw a line that best models the data. The line is called the line of best fit or the
regression line or the least squares regression line.

This line can be used to predict/estimate the value of one variable, given the

value of the other variable.

The line of best-fit may not be the same as someone else’s, it is helpful

to have a systematic method that always gives the same result.

One procedure commonly used is the ‘method of least squares’.

The equation of the linear regression line is y=a +bx, where b is the gradient of the line
and a is the cut on the y-axis (or the y-intercept).

67
Using a Least Squares Regression Line to Make Predictions

The required value can be read off from the scatter plot or by using the equation of
the regression line. Predictions made from the equation of the line can be made
through the process of interpolation and extrapolation.

Interpolation is a method of predicting/estimating new data

value(s) within the known range of data values.

Extrapolation on the other hand is a method of estimating new data

value(s) beyond a discrete set of known data values.

Data values that are a result of extrapolation are often less valid than those that are
the result of interpolation.

EXAMPLE

Use your calculator to determine the equation of the regression line for the

following set of data:

Mass (in kg) 49 65 82 60 65 94 88

Height (in cm) 156 176 183 153 163 192 180

SOLUTION:

250

200

150
Mass ( in kg)

100

50

0
40 50 60 70 80 90 100
Height (in cm)

y=0,8125 x +113,47
r =0,8188

68
Question 1

The table below shows the age and foot length of the right foot for a group of 10 boys as
recorded in the 2009 Census@School.

Age (in years) 12 14 13 11 16 19 15 18 11 19

Foot length (in 23,4 25,4 24,8 22,9 26,5 28 24 27 22,9 30


cm)

a) Draw a scatter plot for the given data.

b) Calculate the value of r, the correlation coefficient and use it to describe the correlation that
exists between the two variables

c) Calculate the equation of the regression line. Give your answer to TWO decimal places.

d) Use your regression line to determine:

i) The foot length of a boy 9 years old.

ii) The foot length of a boy who is 17 years and 6 months old.

e) State whether you think each of the predictions in (c) is good or not. Give reasons for your
answer.

Question 2

A training manager wants to know if there is a link between the hours in training (x) spent by a
particular category of employee and their productivity (units produced per day, y) on the job.
The data below was extracted from the files of 10 employees.

Employee 1 2 3 4 5 6 7 8 9 10

Hours in 16 36 20 38 40 30 35 22 40 24
training(x)

Productivity 45 70 44 56 60 48 75 60 63 38
(units ptoduced
per day) (y)

2.1 draw a scatter plot for the data.

2.2 Using the least squares method, establish a linear relationship between training hours and
productivity for these employees.

2.3 Draw the least squares line for the data on the scatter plot diagram

2.4 Estimate the productivity level for a particular employee who has received only 22 hours of
training.

2.5 Determine the correlation between productivity and hours of training.

2.6 Is the association strong? Advise the manager.

Question 3

69
During the month of July, a number of patients visited a local clinic suffering from
influenza. The table below shows the cumulative number of patients treated as per
the dates given.

Dates in the 3 5 8 12 15 19 22 26
moth of July

Number of 270 275 376 420 602 684 800 820


patients treated

3.1 Draw a scatter plot of the above data

3.2 Determine the equation of the least squares line for the data.

3.3 Draw the least squares line for the data

3.4 Estimate how many patients were treated as at 30 June.

3.5 Estimate on which day in July were there 600 patients treated.

3.6 Determine the correlation coefficient for the data. Interpret this result

Question 4
The term latitude refers to how far a place is from the equator. Latitude in the Northern
Hemisphere range from 0° at the equator to 90° N at the north pole.
Below are the latitudes of several cities in the Northern Hemisphere together with the mean
maximum temperature for April in degrees Celsius.
City Northern Latitude Mean maximum
temperature

Lagos, nigeria 6 32

London, England 52 13

Calcutta, India 23 36

Rome, Italy 42 20

Moscow, Russia 56 8

Cairo, Egypt 30 28

San Juan, Puert Rico 18 29

Copenhagen, Denmark 56 10

Tokyo, Japan 35 17

4.1 Draw a scatter plot for the above information

4.2 Determine the equation of the least squares regression line.

4.3 Draw the least squares regression line on your scatter plot

4.4 What information does the y-intercept of this line represent?

70
4.5 The city of Madrid has a latitude of 40°N.
Determine the mean maximum temperature for April for this city.
4.6 Calculate the correlation coefficient of the data.

4.7 Explain the correlation between latitude and the mean maximum temperature for April.

Question 5
The table below shows the number of hours that a sales representative of a compony
spent with each of the nine clients in one year and the value of the sales(in thousands
of rands) for that client.

NUMBER OF HOURS 30 50 80 100 120 150 190 220 260

VALUE OF SALES (IN 270 275 376 100 420 602 684 800 820
THOUSANDS OF
RANDS)

5.1 Identify the outliers.

5.2 Calculate the equation of the least squares regression line.

5.3 The sales representative forgot to record the sales of one of his client. Predict the
value of this client’s sales(in thousands of rands) if he spent 240 hours with the him
during the year.

5.4 What is the expected increase in the sales for EACH additional hour spent with the
client.

5.5 Draw the least squares regression line for the data on the scatter plot.

Question 6

71
A familiaer question amongst proffessional tennis playersbis whether the speed of the
tennis serves(in km/h) depends on the height of a player (in metres). The heightof 21
tennis players and the average speed of their serve were recoded during a tournament.
The data is represented in the scatter plot below. The least squares regression is also
drawn.

6.1 Write don the fastest average serve speed(in km/h) achieved in this tornament.

Consider the following correlation coefficints:

A. r = 0,85 B. r =-0,5 C. r = 0,5

Which ONE of the given corelation coefficient best fits the plotted data?

6.2 Use the scatter plot and the least squares regression line to motivate

6.3 What does the data suggest about the speed of a tennis serve (in km/h) and the height
of a player (in metres?

6.4 The equation of the least regression line is given as y=27,07+bx . Explain why in this
context, the least squares regression line CANNOT intersect the y-axis at (0;27,07).

QUESTION 7

72
A group of students did some part-time work for a compony. The number of the hours
worked and the payments (in Rands) received for the work done is shown in the table
below. The scatter plot is drawn for the data.

Number of 6 7 8 10 13 15 18 20 23 25
hours worked

Payments ( in 100 120 150 180 250 280 290 320 270 400
Rands) 0 0 0 0 0 0 0 0 0 0

7.1 Calculate the standard deviation of the number of hours worked.

7.2 Determine the number of hours that a student needed to work in order to receive a
payment that was more than ONE standard deviation above the mean.

7.3 Calculate the equation of the least squares regression line.

7.4 Mapula who worked for 11,5 hours was omitted from the original data. Calculate the
possible amount thath the compony has to pay Mapula

7.5 Use the scatter plot to identify an outlier and give the reason for this point to be an outlier.

QUESTION 8

73
Twelve athletes train to run the 100 m sprint event at the locat athletic club trials. Some of
them took their training more than others. The following table and scatter plot shows the
number of days that an athlete trained and the time taken to run the event.The time taken,
in seconds, is rounded to one decimal place.

Number of days 50 70 10 60 60 20 50 90 10 60 30 39
of training 0

Time taken (in 12, 13, 17, 11, 18, 16, 14, 11, 10, 12, 17, 14,
seconds) 9 1 0 3 1 5 3 7 2 7 2 3

8.1 Discuss the trend of the data collected.

8.2 Identify any outlier(s) in the data.

8.3 Calculate the equation of the least squares regression line.

8.4 Predict the time to run 100 m sprint for the athlete training for 45 days.

8.5 Calculate the correlation coefficient.

8.6 Comment on the strength of the relationship between the variables

74
QUESTION 9

The wind speed (in km per hour) and the temperature in (ºC) for the certain town were recorded at
16:00 for a period of 10 days. The information is shown in the table below.

Wind speed (km/h) 2 6 15 20 25 17 11 24 13 22

Temperature(in ºC) 28 26 22 22 16 20 24 19 26 19

9.1 Determine the equation of the least squares regrression line for the data.

9.2 Predict the temperature at 16:00 if, on the certain day , the wind speed of this town war 9 km
per hour.

Interprete the value of b in the context of the data.

RESOURCES FOR MODULE 1

1. 2021 Annual Teaching Plan


2. DBE Mathematics CAPS
3. DBE 2016 – 2021 Mathematics Examination Guidelines
4. Siyavula Everything Maths Grade 10 and 11

MODULE SUMMARY
In this module, participants:
 Defined a mean, median, mode, range, standard deviation, line of best fit and correlation
coefficient.
 Calculate the mean , median, mode , range ,standard deviation, and least regression line
 Describe the relationship that exists between the variables in the scatter plots .
 do more work with statistics
 Practice graph interpretation.

END OF MODULE

MODULE 2: PROBABILITY

75
INTRODUCTION

In this module participants will look at the concept of probability with specific focus on:
 Determining the probability of events

 Using different ways to represent the sample space.

OVERVIEW

In this topic, participants will work with probability in familiar and unfamiliar contexts

SPECIFIC OBJECTIVES
When participants complete this module, they should be able to:
 Determine the probability using the identity , complementary event and mutually exclusive
events
 Determine and calculate the probability using tree diagram, contingency table and
determining independent and dependent events
 Determine and calculate probability using counting principles.
GLOSSARY OF TERMS
Probability chance / likelihood of an event happening

Trial the attempt made to see if an event will occur


eg: each toss of the coin

Outcome result of the trial – (all the elements)

Event collection of one or more outcomes that you are


interested in / subset of sample space

Set Collection of items, labeled with a capital letter

Sample set of all possible outcomes.


space Represented as:
1. set-notation A= {1, 2, 3}

2. tree-diagram
A
3. Venn diagram

4. in a table – contingency tables

Random cannot be predicted


Probability Probability = number of favourable outcomes
total number of outcomes
P(A) = n( A)
n( S )
n(A) number of outcomes in event A
(number of ways A can take place)

n(S) number of outcomes in the sample space


(total number of possible outcomes)
P(A)
the probability of A happening (A taking place)

CONTENT

76
Participants will study this module through the following units

Unit 1 Complementary events, mutually exclusive and use of Identity in Probability

Unit 2 Tree diagram, Contingency table and independent events

Unit 3 Fundamental Counting Principle

UNIT 1: Mutually exclusive events, Complementary events, Venn diagram and the use of identity

INTRODUCTION

In this unit participants will look at the concept of mutually exclusive events, complimentary events,
Venn diagrams and the use of the identity
LEARNING OUTCOME

At the end of this Unit, participants should be able to:


 Determine and calculate the probability of mutually exclusive events
 Determine and calculate probability of complimentary events
 Determine and calculate the probability using the identity

LESSON NOTES

Probability is the chance that something will happen - how likely it is that some event will happen.
Sometimes you can measure a probability with a number like "10% chance of rain", or you can use
words such as impossible, unlikely, possible, even chance, likely and certain.

Example: "It is unlikely to rain tomorrow".

PROBABILITY SCALE

77
Venn diagrams are used to show how events are related to one another. Venn diagrams
can be helpful when doing calculations with probabilities
SHAPES USED IN VENN DIAGRAMS
• Usually a RECTANGLE is used to represent a sample space (S). However, any
closed shape could be used. S

• Usually CIRCLES are used to represent events within the sample space. Again,
any closed shape could be used. S
A B

Term Explanation Venn Diagram

Intersection( and) The intersection of A and B


is an event consisting of
common outcomes that are
in A and B.

Union(or) The union of P and Q is an


event consisting of all
outcomes that are in P or Q

Inclusive Events These are events which do


have elements in COMMON

Exhaustive Events Cover ALL the elements of


6
the sample space .
1 3

78
MUTUALLY EXCLUSIVE EVENTS

Mutually exclusive events are events that cannot happen at the same time. The occurrence of one
event excludes the possibility of the other event occurring.

Examples of mutually exclusive events:


 Turning left and turning right are mutually exclusive because you can’t do both at the same time.
 A learner cannot be in Gauteng and in Western Cape at the same time.
 A learner cannot be in Grade 11 and in Grade 12 at the same time.

Examples of events that are NOT mutually exclusive:


 A learner could live in Benoni and be in Grade 10, so living in Benoni and being in Grade 10 are
NOT mutually exclusive.
 Turning left and scratching your head can happen at the same time, so they are NOT mutually
exclusive
 Taking a King (K) and taking a heart ( ) from a pack of cards in NOT mutually exclusive as the

card could be the king of hearts (K).

USING VENN DIAGRAMS TO ILLUSTRATE MUTUALLY EXCLUSIVE EVENTS AND EVENTS THAT ARE
NOT MUTUALLY EXCLUSIVE
S
Mutually exclusive sets are disjoint sets.
If events A and B are mutually exclusive, then A B
 P(A and B) = 0
 P(A or B) = P(A) + P(B)

If events are NOT mutually exclusive, then A B


 P(A and B) ≠ 0
 P(A or B) = P(A) + P(B) – P(A and B)

A and B

THE ADDITION RULE:


P (A or B) = P (A) + P (B) is called the addition rule for mutually exclusive events.

Because P (A and B) = 0.

79
NB
When events are mutually exclusive we can determine the probability of one event or the other
occurring by adding the probabilities of each happening.

IDENTITY

IDENTITY

P (A or B) = P (A) + P (B) – P (A and B)

S
A B
As can be seen by looking at the Venn diagram alongside, we
need to subtract P (A and B) from A and B, otherwise it will
be included twice.

COMPLEMENTARY EVENTS

Complementary events are events that cannot occur at the same time. If the event A occurs, then
the complement of A is not A. The complement of A is also written A' .

Examples of complementary events are


 Having English as a home language and not having English as a home language.
 Tossing a coin and getting a head or not getting a head.
 Rolling a fair dice and scoring a 3 or not scoring a 3.

Examples of events that are not complementary:


 Rolling a 5 on a dice and rolling a 3 on a dice.
It is not certain that the dice will only land on a 5 or a 3. It could also land on 1, 2, 4 or 6.
 Randomly selecting a learner who lives in a house and randomly selecting a learner who lives in
a traditional dwelling.
It is not certain that a learner lives only in a house or a traditional dwelling. They may also live in
a flat, townhouse, a room at the back of a house, etc.

When events are complementary, the event either happens (A) or does not happen (not A).
 It is certain that they will happen, or not happen, so P (A) + P (not A) = 1.
 Another way to say this is P (not A) = 1 – P (A) or P (A) = 1 – P (not A).

80
Complementary events can also be shown on a Venn diagram
S

A not A

ACTIVITY 1.1

1. A fair die is rolled: Event A represents the die landing on an even number and

Event B represents the die landing on a number greater than 3

Sample space: S = …………………………….

Event A: A = …………………………….

Event B: B = …………………………….

Then A and B = …………………..

2. A sample set consists of natural numbers less than 13.

Event C consists of the factors of 6 and

Event D consists of the factors of 9

Sample space: S = …………………………….

Event C: C = ……………………………..

Event D: D = ………………………………

Then C and D = ………………………

3. A die is rolled. Event A consists of even numbers

Sample space: S = …………………….

Event A: A = ……………………..

Then not A = A/ = ………………..

81
4. A box contains 5 red smarties, 2 green smarties, 3 pink smarties and 4 blue smarties.

What would the complement be of?

a) A if A contains red and green smarties?

b) B if B contains blue and green smarties?

c) C if C contains green, red and blue smarties?

ACTIVITY 1.2

1. Suppose a fair die is rolled and the following events are defined:

A = {2, 3, 4, 6} and Z = {2, 6}

The corresponding Venn diagram is:

This diagram is useful to verify the following:

a) Sample space: S = ………………………………

b) A and Z = ……………………………………….

c) A or Z = ……………………………..

d) not A = ……………………………………….
= ………………………………………………..
e) not (A or Z)

82
2. A die is rolled. Event A are even numbers and Event B are numbers greater than 3

a. List the following: Sample set = …………………………………...

Event A = …………………………………….

Event B = ………………………………………...

b. List the following: n(S) = …………………….

n (A) = ………………...

n (B) = ……………….

c. List the following: P(S) = ……………………………………………………………………

P (A) = ……………………………………………………………………

P (B) = …………………………………………………………………..

d. Draw a Venn-diagram for both events

e. Determine the following using the Venn- diagram

n( A)
=
(i) P(A) = n( S)

n(B)
=
(ii) P(B) = n(S)

n( A and B )
=
(iii) P(A and B) = n( S)

n( A or B)
=
(iv) P(A or B) = n( S)

n(not A )
=
(v) P(not A) = n(S)

83
3. A bag contains 6 blue marbles, 5 red marbles, 8 green marble and 9 white marbles.

Determine the probability of the following:

a. Drawing a white marble? ………………………………………

b. Drawing a green marble? ………………………………………

c. Drawing a blue marble? …………………………………………

d. Drawing a red marble? ………………………………………….

e. Drawing a red or a blue marble? ………………………………….

f. Drawing a blue or a green marble? …………………………………..

g. Drawing a pink marble? ………………………………………………

h. Drawing a white, green or red marble? ……………………………….

i. Not drawing a blue marble? ……………………………………………….

ACTIVITY 1.3

A. Decide whether the following events are mutually exclusive or not

1. Event A: roll a 3 on a die

Event B; roll a 4 on a die

2. Event A: randomly select a male student

Event B: randomly select a nursing major

3. Event A: randomly select a blood donor with type O blood

Event B: randomly select a female blood donor

4. Event A: randomly select a vehicle that is a Ford

Event B: randomly select a vehicle that is a Toyota

B. Determine as required

1. You select a card from a standard deck. Jokers are excluded from a deck of cards.

Determine the probability that the card is a 3 or a king

P(3) +P(K) =

84
2. You roll a fair die.

Determine the probability of rolling a number less than three or rolling an even number.

P(less 3) +P(even numbers) =

3. A fair die is rolled. Find the probability of rolling a 4 or an odd number.

P(4) +P(odd numbers) =

C. Answer TRUE or FALSE: If two events are mutually exclusive, they have no outcomes in common.

1. Event A: randomly select a female worker

Event B: randomly selects a worker with a college degree

Event A and B are …………………………………………………

2. Event A: randomly select a person between 18 and 24 years old

Event B: randomly select a person between 25 and 34 years old

Event A and B are ………………………………………………….

D.A blood bank catalogues the types of blood, including positive or negative Rh-factor, given by
donors the last 5 days. The number of donors who gave each blood type is shown in the table below:

Blood type

O A B AB TOTAL

Positive 156 139 37 12 344

Rh-factor Negative 28 25 8 4 65

TOTAL 184 164 45 16 409

A donor is selected at random. Determine the probability that the donor has :

a) Type O or Type A blood: P(type O or type A = P(type O) + P(type A)

b) Type B blood or is Rh-negative : P(Type B or Rh-negative)

c) Type B or Type AB blood : P(Type B or Type AB)

85
E. A card is selected from a standard deck of cards. Find the probability of the following

a) Randomly selecting a heart or a 3

b) Randomly selecting a black suit or a king

c) Randomly selecting a 5 or a face card

F. A fair die is rolled. Find the probability

a) Rolling a 6 or a number greater than 4

b) Rolling a 3 or an even number

86
UNIT 2: Venn diagrams, Tree diagram, Contingency table and independent events

INTRODUCTION

In this unit participants will look at the concept of mutually exclusive events, complimentary events,
Venn diagrams and the use of the identity
LEARNING OUTCOME

At the end of this Unit, participants should be able to:


 Determine and calculate the probability of Independent events
 Determine and calculate probability using a contingency table
 Determine and calculate the probability using tree diagram and Venn diagrams

LESSON NOTES

VENN DIAGRAM

THREE EVENTS

How to complete the outcomes of the Venn diagram having three events

1.Complete the All first: The intersection of all three events

2.Complete the Pair : The intersection of two events : Subtract the intersection of all Three

3.Complete the Individual : Complete in the individual event

4. Complete the Outside: Complete in the outcomes of the outside

Start completing with the intersection of ALL


events

87
Second complete the PAIR intersection of two
events

Thirdly Complete the Individual the individual


event

ALL intersection
PAIR
INDIVIDUAL
OUTSIDE

COMPOUND EVENETS

 In some probability problems we look at the combined or compound outcomes of several


activities. These are example of activities :
- Shoe size and age
- Spinning a spinner and taking a card
- Throwing two dice
- Tossing a coin three times.

To find out the possible outcomes (the sample set), you can make a Venn diagram, a list, draw up a
table, or use a tree diagram. It is best to use a table or a tree diagram to make sure that you do not
miss any outcome

88
INDEPENDENT AND DEPENDENT EVENTS

INDEPENDENT EVENTS

DEFINITION

Two successive events A and B are said to be independent if the outcomes of the first event does
not influence the outcomes of the second event .

INDEPENDENT EVENTS FROM DIFFERENT INDEPENDENT EVENTS FROM REPEATED


ACTIVITIES ACTIVITIES

When two events result from two completely Repeating an activity in exactly the same way
separate and different activities, it is clear that and under the same circumstances results in
they will be independent. independent events.

Here are some examples of different activities: Here are some examples of repeated activities:

Tossing a coin and Tossing a coin twice


taking a card OR tossing two coins

Throwing a dice
Throwing a dice and
several times OR
spinning a spinner.
throwing several dice

If 2 events are independent then the PRODUCT RULE: can be used

P (A and B) = P (A) x P (B)

DEPEDENT EVENTS:

DEFINITION

Two successive events are dependent, if the outcomes of the first event do have an influence on the
outcomes of the second event.

DEPENDENT EVENTS

Suppose that a lunch box contains four sandwiches, grapes


and nuts.

Event A is picking an item of food from the box and eating


it.

Event B is picking an item from the box again and eating it.

Clearly event B depends on what happened in event A

89
Picking a ball from a jar without replacing

.
CONTINGENCY TABLE

CONTINGENCY TABLE

DEFINITION:

A two—way contingency table shows the observed frequencies for two variables in various
categories. Each variable has two categories (choices) which are mutually exclusive (events that do
not occur at the same time) and exhaustive (the categories cover all possibilities so there is a
category for everyone.

Suppose that a survey was performed to investigate the size of farms and age of farmers. The
results are presented in the following 3 x 5 contingency table.

Farm size Up to 39 40 – 49 50 – 59 60 – 69 70 and over

Small 73 64 59 39 20

Medium 42 69 108 60 21

Large 5 18 85 120 22

The above table is an example of a 2-way contingency table as it has 2 variables, namely farm size
and age

The one variable (farm size) has 3 categories, and the other variable (age) has 5 categories

TREE DIAGRAM

The probability of a complex event can be found using a probability tree diagram

1. Draw the appropriate tree diagram.


2. Assign probabilities to each branch. (Each branch sums to 1.)
3. Multiply the probabilities along individual branches to find the probability of the outcome at the
end of each branch.
4. Add the probabilities of the relevant outcomes, depending on the event.

90
E.g. If a coin is tossed three times

1st toss 2nd toss 3rd toss Outcomes


H HHH
H
T HHT
H
H HTH
T
T HTT

H THH
H
T THT
T
H TTH
T
T TTT

ACTIVITY 2.1

A. Decide whether the events are independent or dependent

1. Selecting a king from a standard deck (A), not replacing it, and then selecting a queen from the
deck (B)

2. Tossing a coin and getting a head (A), then rolling a six-sided die and obtaining a 6 (B)

B. Determine as required

1. A coin is tossed and a die is rolled. Determine the probability of getting a head and then rolling
an even number

91
ACTIVITY 2.2

1.If P(A) = 0,3 and P(B) = 0,5 and P(A or B) = 0,7, Complete the Venn diagram

Determine :

1.1) P (A and B)

1.2) P (not A and B)

1.3) P (not A)

1.4) P (not A or not B)

2. Events A and B are mutually exclusive with P(A) = 0,3 and P(A or B) = 0,7

2.1) Are A and B complementary?

2.2) Find P (B)

3. A boy has 0, 19 probability of winning an art competition and a 0, 13 probability of winning a


talent competition where he will perform a song. Suppose he has a 0, 11 chance of winning both
the art competition and the talent competition?

3.1) Define events

A: The boy wins the art competition: P (A) = ………………………..

T: The boy wins the talent competition: P (T) = ………………………….

P (A and T) = …………………………

3.2) Are events A and T defined in (a) mutually exclusive?

a) What is the probability that the boy does not win the art competition?

b) What is the probability that he wins the art competition or the talent competition?

92
Activity 2.3

1. The South African Demographic and Health Survey was carried out in 1998. In part

of the survey, 5671 men were asked about their drinking habits. Their blood

pressure was also measured. The results are summarized in the following

contingency table.

Number of men High blood pressure Blood pressure not Total


high

Drink alcohol 688 1864

Don’t drink alcohol 611 2508

Total

a) Complete the table above by filling in the missing totals

b) How many men had high blood pressure?

c) How many men drank alcohol?

d) If we assume that the men in the survey are representative of all South African men,

then what is the probability (as a fraction) that a man chosen at random in

South Africa will be:

i) a drinker

ii) a drinker with high blood pressure

iii) a non-drinker with high blood pressure?

e) What does the answer to the last two questions (dii and diii) suggest to you about

the effect of drinking on blood pressure?

2. A survey is done amongst primary school children and the following information

93
was recorded : There are 18 boys in the class of which 10 own a bicycle

There are 10 girls in the class and 4 do not own a bicycle

a. Set up a 2 x 2 contingency table to depict the information above.

b. determine the probability that a randomly selected child is a boy

c. determine the probability that a randomly selected child owns a bicycle

d. determine the probability that a randomly selected child is a boy and owns a bicycle

e. Is the event of owning a bicycle independent of being a boy?

Give reasons for answer

UNIT 3: Fundamental Counting Principle

94
INTRODUCTION

In this unit participants will look at the concept of counting principle and determining the probability
LEARNING OUTCOME

At the end of this Unit, participants should be able to:


 Generalise using fundamental counting principle.
 Probability problems using the fundamental counting principle

Learner Notes

FUNDAMENTAL COUNTING PRINCIPLE

The fundamental counting principle is a quick method for calculating numbers of outcomes
using multiplication.

DEFINITION

• Fundamental: “essential, of central importance”


• Counting: “to determine the total number of…”
• Principles: “underlying rules of the counting principles”

It is possible to list all possible outcomes using a tree diagram. When you have many possible
outcomes, a tree diagram can become very messy and it becomes difficult to count the possible
outcomes.

We therefore using counting principles help count the possible outcomes without drawing a
tree diagram.

COUNTING METHOD 1
Example 1.

95
In a debating committee election, there are 4 candidates for a chairperson and 2 candidates for
deputy chairperson. How many possible chairperson / deputy chairperson are there.

Label the candidates for chairperson as R, P, M and Y and label the candidates of

Deputy Chairperson as X and Z.

1. One way of counting the outcomes is using a tree diagram.

Y
Chairperson R M
P
X Z
Deputy Chairperson X Z X Z X Z

Outcome
MX MZ YX YZ
RX RZ PX PZ
2. Another way of counting the outcomes is by making of a table.

Chairperson

R P M Y

Deputy X RX PX MX YX
Chairperso
Z RZ PZ MZ YZ
n

In both cases there are 8 possible outcomes.

There is another way of getting the total number of outcomes, called Fundamental Counting
Principle.

Counting Principle 4 ×2=8

96
Example 2

1. There are three routes that connect town A and town B and two routes that connect town B
to town C. Use more than one way to determine all possible routes from town A to town C
via town B.
One way

X
1
B
A Y
C
Z 2

Outcomes: X1 X2 Y1 Y2 Z1 Z2 6 outcomes
1 X1
Another way
X
2 X2
1 Y1
Y
2 Y2

1 Z1
Z
2 Z2

Using counting principle, it will be3 ×2=6

Generalise

If there are m objects for the first decision and n choices for the second decision then there are
m× n choices for the first decision followed by the second decision

97
COUNTING METHOD 2

Example

Determine the number of ways a netball team consisting of 7 players can be arrange themselves in a
line for a group picture

The 7 positions can be represented by 7 blanks as follows:

1 2 3 4 5 6 7

There are seven ways to choose from among the 7 players to fill the 1 st position. When the second
position is filled 6 players are left, which give us 6 ways to fill the 2 nd position. Then proceed in the
similar manner.

7 6 5 4 3 2 1
1 2 3 4 5 6 7

Generalize

When arranging n distinct players in a line , the number of different positions we can find is

n × ( n−1 ) × ( n−2 ) … … .× 2×1

If it’s only 4 position for 7 players then

 There are 7 ways of filling the first position


 Having filled the first position then there are 6 ways of filling the second position
 Having filled the 1st two positions , then there are 5 ways of filling the 3 rd position
 Having filled the 1st three position , then there are 4 ways of filling the 4 th which is the last
position

The solution is 7 ×6 × 5 ×4

98
COUNTING METHODS WHEN OBJECTS ARE NOT DISTINCT

Example

Word Different words No. of words No. of identical


letters

BULL BULL;BLUL;BLLU;UBLL 12 2Ls 4!


2!
ULBL;ULLB;LLUB;LLBU

LBLU;LBUL;LULB;LUBL

EZEE EZEE;EEZE;EEEZ;ZEEE 4 3Es 4!


3!

ANAN ANAN;ANNA;AANN;NAN 6 2As and 2Ns 4!


A 2!2 !

NNAA;NAAN

GENERELIZE

The number of distinct n objects of which p are identical to each other and then q of the
n!
remainder are identical then
p !× q!

 Then :
 If there are n different items that are all different,
then there are n × (n – 1) × (n – 2) ... n terms or n! Arrangements.

 If there are n different items, but one item is repeated twice,


n!
then there are n × (n – 1) × (n – 2) ... n terms divided by 2 or 2! Arrangements =
2!
arrangements

 If there are n different items, but one item is repeated three times,
n!
then there are n × (n – 1) × (n – 2) ... n terms divided by 3 × 2 × 1 or 3! Arrangements =
3!
 Sometimes more than one letter is repeated.
 For example, in the word DODO, D is repeated twice and O is repeated twice

 Here you can say that if there are n different items, but two items are repeated twice,

99
n!
then there are n × (n – 1) × (n – 2) ... n terms divided by 2! × 2! Arrangements =
2! × 2!
Arrangements
COUNTING METHOD OF LETTERS WHERE THE WORD START WITH A CERTAIN LETTER AND END
WITH A CERTAIN LETTER

Word No. Of Letters & No. of arrangements Start with and end with
identical letters
n(S)

MISSISSIPPI 11 letters 11! We want Start with an I


=34650
4 ! × 4 ! × 2! and end with an I
4I’s; 4S’s and 2 P’s
I_________I

9!
n(E)= =3780
2! × 4 ! × 2!

3780 6
∴ P ( E )= =
34650 55

STATISTICS 10 Letters 10 ! Start with an I and end


=50400
3! × 3 !×2 ! with an A
3S’s;3T’s &2I’s
I________A

8!
n ( E)= =3360
3 ! ×3 !

3360
P ( E )=
50400

WE SOMETIMES ARRANGE PEOPLE IN A ROW, WITH SOME OF THEM TOGETHER IN EVERY


ARRANGEMENT.

Example

Suppose you have 3 people. An elastic band binds two of them together. How many arrangements
of the three people could be made?

The 3 objects becomes 2 objects. 100

There are 2!2! Ways to arrange them


The NUMBER OF arrangements, WHEN people, books, object etc. are bound by an “elastic band”,
is counted as follows (using examples)

EXAMPLE

When having 50 objects, of which 7 objects and 6 objects are grouped together (bound by elastic
bands), the number of possible arrangements is:

50 is the number of objects and 7 objects to be grouped together then 6 objects to be grouped
together.

Then 50−6−7=37

7 objects grouped together makes 1 group and 6 objects grouped together makes another group

Then 37+1+1=39

n ( E ) =39! × 7 !×6 !

n ( S )=50 !

The probability of having these groups arranged next to each other

39 ! ×7 ! ×6 !
P ( E )= =2,4337 ×10−12
50 !

Example 2

There are 3 different blue books and 2 different red books

a) In how many different ways can the books be arranged?

12 3 4 5

101
5 4 3 2 1

n (books) = n(S) =5!

b) In How many different ways can the books be arranged , if all the blue books should be
placed next to each other

44 1 2 3 5 5 3 2 1 4

3! – 3 objects

1 3 2
2 4 5 3! – Blue books

n(E) = 3!3!

c) In how many different ways can the books be arranged if all the blue books and all the red
books should be placed next to each other

4 5 1 2 3 2!
1 2 3
1 2 3 4 5

1 2 3 1 3 2 2 1 3 2 3 1 3 1 2 3 2 1
3!

102

4 5 5 4
2!
Example n(S) = 2!2!3!
There are3 identical blue books and 2 identical red books on the bookshelf

a) In how many different ways can the books be arranged on the shelf?

1 1 1 5 5

5 4 3 2 1 =5!

5!
n ( S )=
3 !×2 !

b) If all the blue books should be placed next to each other as well as all the red books placed
next to each other, in how many ways can the books be arranged?

1 12 13
1 5 5 5 5 1 1 1

n(S) = 2!

103
COUNTING METHOD ON CODES

Example 1

How many three-digit numbers can be formed with the digits 1; 2; 3; 4; 5; 6; 7; 8; 9

1.1 no digit is repeated


1.2 repetition is allowed

Solution:

Making a three-digit number using the numbers given which are: 9

1.1 There is a choice from 9 digits then after there is a choice from 8 digits then lastly a choice
from 7 digits because repetition is not allowed.

9 8 7

9 × 8× 7=504

1.2 There is a choice from 9 digits then after there is a choice from 9 digits then lastly a choice
from 9 digits because repetition is allowed.

9 9 9

9 × 9× 9=729

104
EXAM TYPE QUESTIONS
Question 1

1.1 A bag contains 7 yellow balls, 3 red balls and 2 blue balls. A ball is chosen at (4)
random from the bag and not replaced. A second ball is then chosen.
Determine the probability that of the two balls chosen, one is red and the other
is blue.
1.2 Learners at a hostel may choose a meal and a drink for lunch. Their selections on a
certain day were recorded and shown in the partially completed table below.
MEAL
TOTAL
SANDWICH (S) PASTA (P)
Fruit Juice (F) a 30 b
DRINK Bottled Water
(W)
TOTAL 200 250
The probability of a learner choosing fruit juice and a sandwich on that day was 0, 48.
1.2.1 Calculate the number of learners who chose fruit juice and a sandwich for lunch (1)
on that day.
1.2.2 Is the choice of fruit juice independent of the choice of a sandwich for lunch on (4)
that day? Show ALL calculations to motivate your answer.

Question 2
Two learners from each grade at a high school (Grades 8, 9, 10, 11 and 12) are elected to form
a sports committee.
2.1 In how many different ways can the chairperson and the deputy chairperson of (2)
the sports committee be elected if there is no restriction on who may be
elected?
2.2 A photographer wants to take a photograph of the sports committee. In how many
different ways can the members be arranged in a straight line if:
2.2.1 Any member may stand in any position? (1)
2.2.2 Members from the same grade must stand next to each other and the Grade 12 (3)
members must be in the centre?

Question 3
Bongani and Sihle are learners at the same school. Some days they arrive late at school. The
probability that neither Bongani nor Sihle will arrive late on any day is 0,7.
3.1 Calculate the probability that at least one of the two learners will arrive late on (1)
a randomly selected day
3.2 The probability that Bongani arrives late for school on a randomly selected day (3)
is 0,25, while the probability that both of them arrive late for school on that day
is 0,15. Calculte the probability that Bongani will arrive late for school on that
day.

105
3.3 The principal suspects that the late coming of the two learners is linked. (3)
Determine whether the events of Bongani arriving late for school and Sihle
arriving late for school are statistically independent or not.

Question 4
Given the digits : 3 ; 4 ; 5 ; 6 ; 7 ; 8 and 9.
4.1 Calculate how many unique 5 – digit codes can be formed using the digits above, if :
4.1.1 The digits may be repeated (2)
4.1.2 The digits may not be repeated (2)
4.2 How many unique 3 – digit codes can be formed using the above digits, if :

 Digits may be repeated

 The code is greater than 400 but less than 600


(3)

 The code is divisible by 5

Question 5
5.1 Given : P(A)=0,45 ; P(B) = y and P(A or B) = 0,74.
Determine the value(s) of y if A and B are mutually exclusive. (3)
5.2 An organisation decided to distribute gifts bags of sweets to a Grade R class at a
1
certain school. There is a mystery gift in exactly of the total number of bags.
4
Each learner in the class may randomly select two bags of sweets, one after the
other. The probability that a learner selects two bags of sweets with a mystery
7
gift is . Calculate the number of gifts bags of sweets with a mystery gift
118
inside.

Question 6
1 9
Given that A and B are two independent events such that P ( A ∩ B )= and P ( A ∪ B ) = .
13 10
Given that P(A) > P(B) , determine ;
6.1 P(A) (4)
6.2 P(B) (3)

Question 7
The events A and B are mutually exclusive. The events A, B and C are exhaustive. If P(B) = 0,6
and P(C) = 0,3 ;
¿
Determine the range of possible values of P( A) .

Question 8
Given that P ( A ) =0,45 , P ( B )=0,3∧P ( A∨B )=0,615 .
Are the events A and B :
8.1 Mutually exclusive? (3)
8.2 Independent? (3)

106
Question 9
Let A and B be two events in a sample space. Suppose that the P(A) = 0,4 ;
P(B) = k and P(A or B) = 0,7. Determine
¿
9.1 P( A∨B) (1)
9.2 The value of k, for which A and B are mutually exclusive (2)
9.3 The value of k, for which A and B are independent events (4)

Question 10
There are 24 bags of marbles for sale in a shop. In 10 of the bags there are 7 green marbles
and three yellow marbles. The other bag each have x green marbles and yellow marbles.
A bag is chosen at random and a marble is then chosen at random from the bag. The tree
diagram below illustrates the process and the outcomes.

10.1 Determine the values of m and n. (2)


10.2 Determine the value of x . (3)
10.3 What is the probability that a marble chosen is green? (2)

Question 11
A take-away shop used a sample group of 100 matric learners to research favourite take-away
meals. The result is as follows :

 58 learners like burgers

 53 learners like pizza

 38 learners like pasta

 16 learners like pizza and pasta

 x learners like burgers and pizza, but not pasta

 18 learners like burgers and pasta

 10 learners do not eat take-away meals

 5 learners like burgers, pizza and pasta


11.1 Draw a VENN diagram to represent the data. (5)
11.2 Determine how many learners like burgers and pizza, but not pasta (2)

107
11.3 Determine the probability that a learner chosen at random likes only TWO of (4)
the take-away meals

Question 12
A study on eating chocolate and gender yielded the following results.
Eating Not Eating Total
Chocolate Chocolate
Male 45 25 70
Female 35 45 80
Total 80 70 150
12.1 How many people participated in this study?
12.2 Calculate the probability of being Male.
12.3 Calculate the probability of Eating Chocolate.
12.4 Are events being a “male” and “ Eating Chocolate’ independent?
Justify your answer with relevant calculations.

Question 13
A survey was carried out with 240 customers who bought food from a fast food outlet on a
particular day. The outlet sells cheese burgers (C), bacon burgers (B) and vegetarian burgers
(V). The Venn Diagram below shows the number of customers who bought different types of
burgers on the day.

13.1 Are the events V and B mutually exclusive? Give a reason for your answer. (2)
13.2 If a customer from this group is selected at random, determine the probability that he/she did (3)
not buy a vegetarian burger

Question 14
14.1 Three boys and five girls are going for a photo-shoot. They are all seated next to each other
in the same row.
14.1.1 In how many different ways can the entire group be seated? (1)
14.1.2 One boy and girl are a couple and want to sit next to each other. In how many different
ways can the entire group be seated if this couple should sit together? (2)
14.2 Consider the word BELLVILLE. The letters of this word are randomly arranged to form new
words.
Calculate the probability that the word will start with an ‘L’.

Question 15
15.1 A car park has 14 VOLKSWAGEN cars and 18 BMW’s. There are no other cars. During the afternoon

108
two cars are stolen – one early afternoon, the other later. Determine the probability that:
15.1.1 Both cars were BMW’s. (4)
15.1.2 The first one stolen was a BMW and the second one a Volkswagen. (2)
15.2 Eight boys and seven girls are to be seated randomly in a row. What is the probability that:
15.2.1 The row has a girl at each end? (3)
15.2.2 The row has girls and boys sitting in alternate positions? (3)
Question 16
A password consists of five different letters of the English alphabet. Each letter may be
used only once.How many passwords can be formed if:
16.1 All the letters of the alphabet can be used (2)
16.2 The password must start with a 'D' and end with an 'L' (2)

Question 17

17.1 In how many different ways can ALL the cars be parked? (2)
17.2 If the three silver cars must be parked next to each other, determine in (3)
how many different ways the cars can be parked

RECOURCES FOR MODULE 2


1. DBE2019 June Paper 1 Question 10 and 11

2. DBE 2018 Feb, June and November NSC Paper 1Question 11, Question 10

3. DBE 2016 - 2019 November Paper 1 Question 10

4. Siyavula Grade 10 - 12

109
MODULE 3: Financial Mathematics

INTRODUCTION

One of the most common misconceptions found in the Grade 12 examinations is the lack of
understanding that learners have from the previous grades (Grades 10 and 11) and the lack of ability
to manipulate the formulae. In addition to this, many learners do not know when to use which
formulae, or which value should be allocated to which variable. Overall aim of the module: Financial
Mathematics

The study of Financial Mathematics is centred on the concepts of simple and compound growth. The
learner must be made to understand the difference between the two concepts at Grade 10 level.
This may then be successfully built upon in Grade 11, eventually culminating in the concepts of
Present and Future Value Annuities in Grade 12. This section will revise the most important concepts
that lead up to Present and Future Value Annuities.

In this module participants will look at the concept of financial mathematics with specific focus on:
 Determining the simple interest, compound interest.

 Calculating depreciation using reducing balance and straight line

 Annuities

OVERVIEW

In this topic, participants will work with financial mathematics in familiar and unfamiliar contexts

SPECIFIC OBJECTIVES
When participants complete this module, they should be able to:
Learning objectives of the module

110
 To understand and become proficient in the use of different financial mathematics
formulae.

 To understand the difference between present value and future value.

 To understand the difference between simple interest growth and compound interest
growth

 To use simple {𝐴 =P (1 − 𝑖𝑛)} and compound decay { A=P¿ } to solve problems (including
straight line depreciation and depreciation on a reducing balance.

 Link to work on functions.

 To understand the effects of different periods of compounding growth and decay (including
effective and nominal interest rates).

 To apply the knowledge of geometric series to solve annuity and bond repayment.

 To critically analyse different loan options.

Key Content Addressed by this module: ATP (Show curriculum mapping)

CAPS extraction indicating progression from Grades 10-12.

GRADE 10 GRADE 11 GRADE 12

Use simple {𝐴 =P (1 + 𝑖𝑛)} and Use simple {𝐴 = P(1 − 𝑖𝑛)} and Calculate the value of n in the
compound growth { A=P¿ } compound decay { formulae A=P¿
n
formulae to solve problems A=P(1−i ) } to solve
(including interest, hire purchase, problems (including straight
inflation, population growth and line depreciation and Apply the knowledge of
other real-life problems). depreciation on a reducing geometric series to solve
balance. Link to work on annuity and bond repayment.
functions.
The implications of fluctuating
foreign exchange. Critically analyse different loan
The effects of different periods options.
of compounding growth and
decay (including effective and
nominal interest rates).

111
WEIGHTINGS OF CONTENT AREAS

Description Grade 10 Grade 11 Grade 12

Finance and growth 𝟏𝟎 ± 𝟑

Finance, growth and decay 𝟏𝟓 ± 𝟑 𝟏𝟓 ± 𝟑

The Compound Interest Formula The Simple Interest Formula


n A=P(1+in)
A=P(1+i ) A – Accumulated amount. This is the total
A – Accumulated amount. This is the total amount paid after a perion of n years.
amount paid after a perion of n years. It includes the original amount P plus interest.
It includes the original amount P plus interest. P – Principal amount. This is the original
P – Principal amount. This is the original amount borrowed or invested.
amount borrowed or invested. n – the period of investment in years.
n – the period of investment in years. i-interest rate per annum
i-interest rate per annum

SIMPLE AND COMPOUND GROWTH

 What is our understanding of simple and compound growth?


 How do we, as educators, effectively transfer our understanding of these concepts to our
learners?
 What do the learners need to know before we can begin to explain the difference between
simple and compound growth?

Example 1 :

R70 000 is taken out to start business. The loan is repaid at the end of 5 years in one lump sum
payment. Calculate how much money is required to repay the loan in each of the following
cases:

1.1 The interest rate is calculated at 12% p.a simple interest

1.2 The interest rate is calculated at 12% p.a compounded annually.

Example 2 :

Sbusiso invests R30 000 in a pension fund for a period of 96 months. The total growth of the
fund accumulates to R65 000 after 96 months.

2.1 Calculate the interest rate per annum compounded annually that would yield the same
return.

2.2 Calculate the simple interest rate per annum that would provide the same return.

112
SOLUTIONS:
1.1
A=P(1+in)
=70000(1+0.12×5)
=R 112000

A=P(1+i )n
=70000(1+0 .12 )5
=R 123 363,92

1.2.

2.1 A=P(1+i)n 2.2 A=P(1+¿)

65000=30000 (1+i )8 65000=30000 (1+8 i )

13 8 13
=(1+i) −1=8 i
6 6


8 13 8
6
= √(1+i)
8
i=14,58333 %

1,101473398−1=i

i=10,15 %

Exercise 1.1.1.

1. R100 000 is invested into a savings account. The amount grows to R250 000 after 10 years.
Calculate the annual interest rate if the interest was compounded annually.
2. Regina buys a TV for R3 500 on a hire purchase agreement. The interest charged is 10%
p.a. based on the full purchase price of the TV. She repays the loan over 3 years. Calculate
the monthly payments she makes to fully pay off the loan.
3. A loan of R9 000 is repaid at the end of 4 years in one amount. Calculate how much money
must be paid if the interest is calculated at 12% p.a. simple interest.

113
Nominal and Effective interest rates

 Interest rate are normally quoted as a percentage per annum.


 Interest can be added more frequently than once a year.
 Interest can be added at any regular period of time during the year.
 The most common time frames are
a. Semi-annually: 2 times a year
b. Monthly: 12 times a year
c. Quarterly: 4 times a year
d. Daily: 365 times a year

EXERCISE1.1.2

 In this activity you will compare the growth of an investment of R100 over a year where
the interest added is compounded every month to an investment of R100 over a year
where interest is only added at the end of the year.
 R100 is invested in a savings account for one year. Calculate how much will be in the
account if interest is added at the end of the year. The interest is 12% p.a.
 Suppose R100 is invested for one year interest is 12% p.a. compounded monthly but
interest is only added at the end of every month. What is the percentage rate of interest
per month?
 Copy and complete the table below
 Record your answers rounded off to the nearest cent in the table, but do not round off
answers on the calculator.

End of month month month month month month

114
Total value
including
compound
interest

End of month month month month month month

Total value
including
compound
interest

 Use the compound interest formula to determine the rate of interest that would yield
R112, 68 at the end of one year.
 Note that the interest calculated above is called the effective interest rate because it is
the actual percentage interest received in one year when the interest was compounded
monthly.

 The quoted interest rate of 12% p.a. compounded monthly is called a nominal rate while the
equivalent annual interest rate is called the effective interest rate.
 The actual interest rate paid was 12, 68% p.a and this is called the effective interest rate.
 We use the symbol i for an effective annual interest rate and I will be used for the nominal
interest rate compounded (m) times per annum

Examples of nominal interest rate


Nominal interest rate interpretation

 12% p.a compounded annually  the interest for the whole year is 12%

 the interest is effective annual interest.

 12% p.a. compounded quarterly  The interest rate for the whole year is 12%
0,12
 The interest rate for a quarter of a year is 4
0,12
∴ i 4=
4

115
 The interest is added four times in a year.

 12% p.a. compounded monthly  The interest rate for the whole year is 12%
0,12
 The interest rate for a quarter of a year is 12
0,12
∴ i12 =
12

 The interest is added 12 times in a year.

 12% p.a .compounded daily  The interest rate for the whole year is 12%
0,12
 The interest rate for a quarter of a year is 365
0,12
∴ i365 =
365
 The interest is added 365 times in a year.

VERY IMPORTANT FORMULAE


Annual Effective Interest Rate Nominal Interest Rate
n m .n
A=P(1+i) im
A=P(1+ )
m

Conversion of Interest rates


2 EFFECTIVE AND NOMINAL INTEREST RATES

 Use the equation to convert from the nominal to

( )
inom m effective rate of interest
1+i eff = 1+
m  This equation is not on the information pamphlet.

 WHEN TO USE: if the rate quoted is per year, and the


payments are in a different periods, then you must
find the effective interest rate FIRST, before doing the
sum.

 WATCH OUT: the rate is 9% per year and payments


are made at the end of every quarter.

 If it says: the rate is 9% p.a. compounded quarterly


and payments are made quarterly ….
then you have nothing to worry about!!!

Worked example: 1.1


1. R8 000 is deposited into a savings account for one year at an interest rate of 13,5% p.a.
compounded quarterly.

116
1.1 What is the nominal interest rate?
1.2 What is the quarterly interest rate?
1.3 Calculate the total amount of money accumulated in the savings account at the end
of 1 year.
1.4 Calculate the effective annual interest rate.

1.1 Nominal interest rate is i 4 =0,135


0,135
1.2 Quarterly interest rate is =0,03375
4
0,135 1 ×4
1.3 A=8000(1+ )
4
A = R 9135,92
1.4 A¿ P ¿
9135,92=8000(1+i)
i=0,141989
i=14,20 %p.a

Exercise 1.1.3

1. Mrs Zuze deposited R240 000 into a fixed deposit savings account for five years. The accumulated
amount at the end of five years is R390 000.
1.1 calculate the effective annual interest rate.
1.2 the nominal interest rate per annum if the interest was compounded monthly.
1.3 the nominal interest rate per annum if the interest was compounded quarterly.

TIMELINES

117
 A time line is a useful way to summarise and represent information in problems concerning investments
and loans.
 Useful in financial mathematics when there are several deposits and withdrawals, as well as possible
changes in interest rates
 Time line provides a visual summary of all the information in an investment fund

Worked Example

Kamohelo opens a savings account with a deposit of R9 400 and two years later a further R5 800 is added to
the savings account. Five years after the savings was opened, another R12 600 is deposited into the account.
The interest paid on the savings is 9% p.a. compounded monthly. Calculate the total amount saved at the end
of eight years.

Solution:

 On the time line


1.1
represents the end of the second year, ***** represents the end of the fifth year and

***** represents the end of the eighth year. **** is the beginning of the first year.
 The time line gives a visual representation of all the information in the problem.

( ) ( ) ( )
8×12 6×12 3×12
0 . 09 0 .09 0. 09
A=9 400 1+ +5 800 1+ +12 600 1+
12 12 12
=R45 681,60

NB: Discuss other methods that can be used to arrive at the above solution.

118
Example *****

 Musa opens a savings account and deposits R31 000. Eighteen months later he withdraws R7 200.

Three years after he opened the account he deposits a further R13 500. One year later he withdraws

R6 700. Calculate the total amount of money in the savings account at the end of six years if the

interest paid on the savings is 10% p.a compounded quarterly.

Solution : Time line

( ) ( ) ( ) ( )
6×4 4,5×4 3×4 2×4
0.1 0.1 0 .1 0,1
A=31 000 1+ −7 200 1+ +13 500 1+ −6 700 1+
4 4 4 4
=R54 833,66

Exercise 1.1.4

1. A savings account is opened with an initial deposit of R7 000. Three years later a second deposit of R3 500
is added to the savings. The interest rate is 8% p.a. compounded quarterly for the first two years. It is then
increased to 9%p.a compounded monthly for the remaining period. Calculate the total accumulated
amount in the account at the end of the five years.
2. Hassan inherits R65 000 from his mother. He invests the money in a savings account. The interest rate is
12,5% p.a compounded monthly for 18 months, and it then increased to 14,2% p.a compounded quarterly.
At the end of two years he withdraws R15 000. Calculate how much he has in his savings account at the
end of five years.
2. Karabo needs R180 000 to start his own business. He is able to obtain a loan from the small business
development fund of from a bank. The loan is repaid at the end of three years in one amount. The small
business development fund charges 8% p.a. simple interest. The bank charges interest at 7%p.a
compounded monthly. Which option will be a better option for Karabo? Justify your answer with

119
calculations.
3. R28 000 is invested is a savings account, and after 15 months R5 500 is withdrawn. Two years after the first
deposit, R15 000 is added to the savings, followed by a further R12 000. Calculate how much is in the
account at the end of four years if the interest rate is 7,5% p.a compounded monthly

Depreciation

INTRODUCTION

 Depreciation is the loss of value of an asset through age. Vehicles machinery and
equipment reduce in value over a period of time and eventually need to be replaced
 There are two types of depreciation. Straight line depreciation and reducing balance
depreciation.
Straight line depreciation:

 Depreciation is the same every year. It is a percentage of the original value of the asset.
 The value of the asset is reduced to zero over a period of time.
Reducing balance depreciation

 Depreciation is based on the previous year’s value.


 Every year the depreciation will be a percentage of the reduced value of asset.
 The loss in value becomes less each year as the asset diminishes in value.
 The value of the asset does not reduce to zero over a period of time.

Straight line depreciation Reducing balance depreciation

Formula: A=P(1−¿) A=P(1−i)n

120
Exercise 1.1.5

1. A company buys some earth moving equipment valued at R860 000. Calculate the value of
the equipment at the end of 6 years if depreciation is calculated at
1.1 14% p.a. on a straight-line basis
1.2 14% p.a on a reducing balance basis
2. A factory buys machinery for R425 700. The machinery depreciates at 20% on a reducing
balance basis.
2.1 Calculate the expected cost of the machinery after 5 years if the inflation is
estimated to be 6.2% p.a.
2.2 Calculate the book value of the machinery after 5 years.
2.3 A sinking fund is set up to replace the machinery at the end of 5 years. R90 000 is
deposited into this fund at the end of each year, starting one year after the
machinery was bought. The last payment is made at the end of the 5 years. Calculate
how much money will be in the sinking fund at the end of the five years if the
interest rate is 10.% p.a.

121
Annuities______________________________________________________________________

 An annuity is a number of equal payments made at regular intervals of time, and subject to a
rate of interest rate. There are two types of annuities:

 Future value annuity: Capital is accumulated by means of regular equal payments into a
savings account or an investment fund. Compound interest is paid on the money deposited
into the fund

 Present value annuity: Money is repaid by regular equal payments in order to pay back a
loan, together with interest accrued.

Future Value Annuity Present Value Annuity


x [ (1+i )n −1 ] x [1−(1+i ) ]
−n

F v= Pv =
i i

 v
F  v
P
- future value - present value
 x - regular equal payments  x - regular equal payments
 i - interest rate  i - interest rate
 n - number of payments  n - number of payments

 Both the future value annuity formula and the present value annuity formula are derived
from the sum to infinity of a geometric series.
 Future value calculations are used when a certain amount of money is placed into an
account on a regular basis in order to save money.
Future value calculations are used for:
1.Retirement annuities
2.Savings Account
3.Sinking Fund
Present Value calculations are used when payments are made on a regular basis in order to
pay back a loan.
Present value calculations are used for:
1.Loan
2.Bond Payments

122
Worked example

1. Bongiwe has R600 deducted from her salary at the end of every month for her pension.
Calculate how much money she will have in her fund at the end of 12 years if the growth is
10% p.a. compounded monthly.
Solution

x [ ( 1+i )n−1 ]
F v=
i

¿
[(
600 1+
12 )
0 ,10 144
−1
]
0 , 10
12
¿ R165 852,73
2. Musa wants to save R800 000 in ten years. She opens a savings
account and pays money into the account at the end of every month.
The interest paid on the account is 8,5% p.a. compounded monthly.
Calculate how much she must deposit each month so that she will
have R800 000 after her last payment.

123
x [ ( 1+i )n−1 ]
F v=
i

[( ) ]
120
0 , 085
x 1+ −1
12
800000=
0 ,085
12
x=R 4 252,19

Loans and Loan repayments


1. Nkululeko buys a house and takes out a loan of R1 000 000. He repays the loan over 25 years. The
interest on the loan is 11% p.a. compounded monthly.
1.1 Determine the monthly repayments on the loan. Assume that the first payment starts one month
after the drawing of the loan.
1.2 Calculate the total amount of money required to repay the loan.
Solution

x [ 1−( 1+i)−n ]
Pv =
i

[( ) ]
−300
0, 11
x 1− 1+
12
1 000 000=0, 11
12

x=R 9 801,13

124
LOANS WITH DEFERRED PAYMENTS

Example 1
Reuben takes out a loan of R200 000 to buy a grinding meal . He makes a total of 238 equal payments
at an interest rate of 12% p .a. compounded monthly starting three months after the loan is granted.
Determine his monthly payments.

Loan is taken out at T 0


N o payment was made at T1 and T 2 .
Loan gains interest for two months .
A=P(1+i )n
A=200000 1+ (0 .12 2
12 )
A=R 204020 .
The first payment is at T 3 and the last one is at T 240 . A total of 238 repayments
x [ 1−( 1+i ) ]
−n
P=
i

[ ( ) ]
−238
0 .12
x 1− 1+
12
204020=
0. 12
12
x=R 2251. 01

Example 2:
A loan of R150 000 is taken out at an interest rate of 7% p .a compounded monthly . Payments start
6 months after the loan was taken out and it is repaid by means of 60 equal monthly payments .
a) Calculate the outstanding balance 5 months after the loan is taken out .
b) Determne the monthly repayment required to payback the loan .

125
Solution:
(a)
Loan gains interest for 5 months
n
A= P ( 1+i )

( )
5
0.07
A=150 000 1+
12
A=R154426.34
∴Outstanding balance is R154426 .34
(b)
Monthly repayments calculations
x [ 1−( 1+ i ) ]
−n
Pv =
i

154426 .34=
[ (
x 1− 1+
0 .07 −60
12 ) ]
0 . 07
12
x=R 3057 .83

CALCULATION OF OUTSTANDING BALANCE

Example 3

A loan of R1 000 000 is taken from a bank to buy a house. The loan is repaid over a period of 20 years and
the interest charged on the loan is 9 % p.a. compounded quarterly .
(a) Calculate the quarterly repayments .
th
(b) Calculate the outstanding balance on the loan after 7 years immediately after the 28 payment .
(c) How much money has been paid at the end of 7 years .
(d) How much money of the actual capital has been paid on the loan at the end of the seven years.
(e) How much of the money paid at the end of 7 years was interest?
Suggested Solution

126
The loan amount is P=R1 000 000
0. 09
The interest is i=
4
The number of payments is n=20×4

1000000=
[ ( )]
x 1− 1+
0 . 09 −80
4
0 . 09
4
x=R 27063.76
(b)
Method 1 Using the Present value annuity formula

When we use the present value formula, the outstanding balance is calculated by looking at all the payments
that still have to be made and bringing them back to the present value at T 28 .
x [ 1−( 1+i ) ]
−n
OB=
i

[( )]
−52
0 ,09
27063,76002 1− 1+
4
OB=
0 , 09
4
OB=R824641,07
Method 2: Using the future value annuity formula

127
We take loan together with the accumulated interest at T 28 and then subtract the repayments together with
accumulated interest at T 28 .
n x [ ( 1+i )n −1 ]
OB=P ( 1+i ) −
i

[( ) −1]
28
0 , 09
27063 , 76002 1+
4
( ) − 0 , 09
28
0 .09
OB=1000000 1+
4
4
OB= R824641,07
(c)
Money paid at the end of 7 years

R27063,76× 28 quarters
=R757785,28
(d)
The actual amount paid off the capital : R1000 000− R824641,07
=R175358,93
(e)
The amount of interest paid in 7 years : R757785,28− R175358,93
=R582426,35

FINAL PAYMENT LOANS:

A loan of R70 000 is repaid by means of equal monthly instalments of R 4000 per month, starting one month after the
loan was drawn. The interest on the loan is 11% p.a. compounded monthly.
(a) Calculate how many monthly payments it will take to repay the loan.
(b) Calculate the final payment on the loan
(c) Calculate how long it would take to repay the loan if the monthly instalments was R350 per month.
Suggested solution:
(a)

128
x [ 1−( 1+i )−n ]
P=
i

[ ( )]
−n
0 ,11
4000 1− 1+
12
70000=
0 , 11
12
77
480 ( )
=1−
1211 −n
1200

( ) ( )
−n
1211 403
=
1200 480

( ) ( )
−n log
1211
1200
=log
403
480
∴n=19 , 1617873
This means that there are 19 full payments of R4000 and one final payment at the end of the 20 th month
which will be less than R4000 .

129
(b)

To calculate the final payment we need to calculate the outstanding balance at the end of the 19 th month .
Outstanding balance = Loan plus interest at T 19− Repayments together with accumulated interest .

[( ) −1]
19
0 ,11
4000 1+
12
( ) −0 ,11
19
0 ,11
OB=R 70000 1+
12
12
OB=R 643 ,72 . ..
This amount is paid at the end of at the end of the 20 th month and therefore accumulates one month's interest .
Hence payment at T 20=643 .73 1+
0 ,11
12 ( )
=R 649 , 63
Alternatively
Outstanding Balance : We use the Pv : We look at all the payments that still have to be made and bring them back
to the present value at T 19 .
x [ 1−( 1+i ) ]
−n
OB=
i

[( ) ]
−0 ,1617873
0 , 11
4000 1 − 1+
12
=
0 .11
12
OB = R643,72. . ..
Hence payment at T 20=643. 73 1+ ( 0 ,11
12 )
=R 649 , 63

(c)

Suggested solution

x [ 1−( 1+i )−n ]


P=
i

[ ( ) ]
−n
0 , 11
350 1 − 1+
12
70000=
0 ,11
12

130
11
6
=1− ( )
1211 −n
1200
−n log ( ) ( )
1211
1200
=log −
5
6

( ) 5
Note that log − is undefined. This means that there is no possible value of n.
6
The monthly interest is greater than the monthly repayment .
The loan will never be repaid .

LOAN OPTIONS

Factors that need to be taken into account when comparing loan options:

 Total repayments

 Duration of payment

Group Discussion

Your friend needs to get a loan to buy a house. The following options are provided by two different banks. Both banks
have a 20 year repayment period.

Which option should your friend choose.

Conditions Bank 1 Bank 2

Loan Amount R600 000 R600 000

Repayments Quarterly payments Monthly payments

Interest rate 13%p.a. compounded quarterly 12.86%p.a compounded monthly

131
EXAM TYPE QUESTIONS

1.1 Calculate the effective rate if a sum of money is invested at a nominal 4


rate of 10, 25% p.a. compounded quarterly.
1.2 Thato wants to go on holiday in 3 years’ time. Her father gives her 8
R25 000 to open a Savings Account. She invests this amount in an
account which pays an interest rate of 7,5% p.a. compounded monthly
1 1
1 1
for 2 years. At the end of the 2 years she deposits an additional
R15 000 into the account. The interest rate for the remainder of the
period is 9, 7% p.a. compounded half yearly.
How much money will Thato have available when she goes on her
holiday?

1.3 Jojo, a taxi owner, buys a new taxi for R280 000.

1.3.1 Calculate the value of the taxi after 4 years if the rate of 4
depreciation is 18% p.a. based on reducing
balance
1.3.2 Calculate the estimated cost of a taxi in 4 years’ time if the expected 4
inflation rate is 6,9% p.a.
1.3.3 How much additional money will Jojo need after 4 years if they 2
sell the taxi at book value and use this money
towards the purchase of a new taxi.

2 Sheena receives R1 500 as a gift. She invests her money in a savings account,
earning interest at 15% per annum compounded semi-annually
2.1 How much money does Sheena have in her investment account at the end of 5 4
years?
2.2 Disa also receives R1 500, but she invests her money in an account which earns 3
interest annually. If Sheena and Disa have the same amount of money at the
end of 5 years, what annual interest rate is Disa earning?
3 A company buys new machinery at the beginning of 2013 for R23 000. The
machinery depreciates on the reducing balance method at a rate of 13, 5% per
annum.
3.1 Determine the book value of the machinery at the end of 2017. 2
3.2 Determine the expected cost of purchasing new machinery at the beginning of 2
2018 if the purchase price at the beginning of 2013 increases at 6,6 %
compounded annually.
3.3 How much would the company have had to invest as a lump sum at the 6
beginning of 2013, if they wanted to pay cash for the new machinery at the
beginning of 2018 and the money is invested in a bank account earning interest
of 4,7% p.a., compounded monthly?

132
4.1 A tractor bought for R120 000 depreciates to R11 090, 41 after 12 years by using the 4
reducing balance method. Calculate the rate of depreciation per annum. (The rate was
fixed over the 12 years.)
4.2 Calculate the effective interest rate if interest is 9, 8% p.a., compounded monthly. 3
4.3 Mrs Pillay invested R80 000 in an account which offers the following: 4

 7,5 % p.a., compounded quarterly, for the first 4 years and thereafter
 9,2% p.a., compounded monthly, for the next 3 years

Calculate the total amount of money that will be in the account at the end of 7 years
if no further transactions happen on the account.
4.4 Exactly 8 years ago Tashil invested R30 000 in an account earning 6, 5% per annum,
compounded monthly.
4.4 How much will he receive if he withdrew his money today? 3
.1
4.4 Tashil withdrew R10 000 three years after making the initial deposit and re-invested R10 000 five years
.2 making the initial deposit.

Calculate the difference between the final amount Tashil will now receive after eight years and the amoun
would have received had there not been any transactions on the account after the initial deposit.

A school buys tablets at a total cost of R140 000. If the average rate of inflation is
5.1 6, 1% per annum over the next 4 years, determine the cost of replacing these tablets
in 4 years’ time.
5.2 An investment earns interest at a rate of 7 % per annum compounded semi-
annually. Calculate the effective annual interest rate on this investment.
5.3 A savings account was opened with an initial deposit of R24 000. 18 months later R7
000 was withdrawn from the account. Calculate how much money will be in the
savings account at the end of 4 years if the interest rate was 10,5% p.a.
compounded monthly.
5.4 A car, costing R198 000, has a book value of R102 755, 34 after 3 years. If the value
of the car depreciates at r % p.a. on a reducing balance , calculate r.

133
6.1 The cost price of a car is R635 000. The value of the car depreciates
according to the reducing balance method at a rate of 15% p.a. Calculate
the value of the car exactly 5 years after it was bought.
6.2 A loan of R50 000 is to be repaid in 48 equal monthly payments. Payments
start one month after the loan has been granted. The interest rate on the
loan is 16, 75% p.a. compounded monthly.
6.2. Calculate the value of the equal monthly payments.
1
6.2. Calculate the outstanding balance immediately after the 30th payment
2 has been made. Give your answer to the nearest Rand.
6.2. How many years will it take for an investment to double its value if the
3 Compound interest rate is 14, 75% p.a? Leave your answer correct to one
decimal digit. (4)

7 A business installs a server for R500 000. The value of the server
depreciates at 20% per annum according to the diminishing-balance
method.
7.1 Calculate the scrap value of the server at the end of 6 years.
7.2 The server needs to be replaced after 6 years.
Calculate the cost of the new server if the inflation rate is at 7% per annum.
The older server will be traded in.
7.3 On the day the server gets installed, the business sets up a sinking fund
into which equal monthly instalments must be paid. Interest on this fund is
8% per annum compounded monthly. The first payment will be made
immediately and the last payment will be made at the end of the 6 year
period. Calculate the value of the
monthly instalment into the sinking fund.
7.4 The business decides to rather pay a monthly instalment of R 15 000 into
the
sinking fund. After how many months will there be more than R1 000 000
in the fund?

8.1 A car, bought for R128 000, depreciates annually at a compound rate. After 6
years it is worth R45 500. At what rate did the value depreciate?
8.2 Keith sold his house for R250 000 and invested the money at 9, 5% p.a.,
compounded quarterly.
Twelve years later he used the proceeds of the investment to buy
another house for R2 920 000 and obtained a mortgage bond for the
remaining amount. The bond was granted for 20 years at 10, 3% interest
p.a. compounded monthly.
8.2.1 Calculate the value of Keith’s original investment after 12 years.
8.2.2 Determine the value of the bond that Keith obtained.
8.2.3 Calculate the monthly payment that he has to make to pay off the bond.
8.2.4 He paid off the bond in 20 years. How much interest did he pay on the
bond

134
END OF MODULE 3
CONCLUSION

This is the end of the e-SSIP training and hopefully, it was informative and fruitfully. You have been
equipped with the Term 3 subject content and ICT skills. You have been provided with the teaching
tips on how to approach each topic or content simplification. All of the above-mentioned skills will
enable you to manage and enhance teaching, learning and assessment in the classroom effectively.

Please share the knowledge gained with other colleagues within your PLCs.

REFERENCES USED

1. DBE NSC Question Papers, 2016 – 2018


2. Mathematics CAPS
3. 2016 – 19 Examination Guidelines
4. 2021 ADJUSTED ATP
5. Siyavula Grade 10 – 12
6. Census @ school

END OF THE COURSE MANUAL

135

You might also like